You are on page 1of 72

Dear 5th year student

Re: Hematology course

The hematology course has been designed to be interactive and clinically oriented. Your
hematology course consists of didactic lectures, multidisplinary seminars, case scenarios and
practical. Please, read the course objectives prior to starting the course to exactly know what
is required from each of you at the end of the course. Case scenarios supplemented in this
course are meant to enhance your abilities in problem solving: understanding the patient’s
complaints, looking for proper signs of any hematological problem, understand the sequence
of events, formulate a plan, order the proper tests and be able to initiate a therapy and follow
outcome of treatment. To gain the maximum benefit, you are requested to read these case
scenarios upfront write your answer in a separate sheet of paper and be ready to give it to
your tutor upon request. Active participation and discussion is required from every body in
class, your tutor is only a facilitator and not information provider during these sessions. You
will be divided into groups (seminar rooms) and be assigned different tutor each module.
Please, send me your comments any time. This is the only way to improve our teaching
abilities.

Sincerely
5th year hematology course Coordinator

2
INDEX
Sr.
Particulars Page No
No.
1 AIM & OBJECTIVES 5
2 COURSE TIMETABLE 7
3 APPROACH TO ANEMIA 10
4 HEMATOLOGY LABORATORY – NORMAL VALUES 13
5 PRINCIPLES OF HEMATOLOGY TESTS 14
6 SLIDES 17

MODULE I – DISORDERS OF RED BLOOD CELLS

4 NUTRITIONAL ANEMIA 20
5 MEGALOBLASTIC ANEMIA 22
6 FOLIC ACID DEFICIENCY 23
7 ALCOHOL AND HEMATOPOIESIS 24
8 APLASTIC ANEMIA 24
9 THE HEMOLYTIC ANEMIAS 25
10 BLOOD TRANSFUSION PRACTICE 28
11 PROBLEMS FOR TUTORIALS 31

MODULE II- WBC DISORDERS

12 LYMPHADENOPATHY AND SPLENOMEGALY 37


13 BENIGN LEUKOCYTE DISORDERS 38
14 HEMATOPATHOLOGY TESTING 40
15 HODGKIN’S AND NON-HODGKIN’S LYMPHOMAS 42
16 MYELOMA 46
17 LEUKEMIAS 48
18 PROBLEMS FOR TUTORIALS 51
19 MYELOPROLIFERATIVE DISORDERS 54

MODULE III & IV - HEMOSTASIS AND THROMBOSIS

20 BLEEDING DISORDERS 57
21 COAGULATION DISORDERS 61
22 ANTITHROMBOTIC AGENTS 66
23 PROBLEMS FOR TUTORIALS 68

3
PROBLEMS FOR TUTORIALS
Sr. Page
Particulars
No. No

MODULE I – DISORDERS OF RED BLOOD CELLS

1 ANEMIA AND CONFUSION 31


2 ANEMIA AND CONFUSION IN ALCOHOLIC 31
3 ANEMIA AND RETICULOCYTOSIS 32
4 ANEMIA AND ARTHRITIS 32
5 ANEMIA AND HEART DISEASE 32
6 ANEMIA, WEAKNESS AND JOINT PAIN 33
7 ANEMIA AND MACROCYTOSIS 33
8 FEVER, ANEMIA AND RETICULOCYTOSIS 33
9 CHRONIC FATIGUE AND ANEMIA 34
10 A YOUNG PATIENT WITH RECURRENT BONY ACHES 35

MODULE II- WBC DISORDERS

1 ADULT MAN WITH LYMPHOCYTOSIS 51


2 A YOUNG FEMALE WITH NECK LUMP 51
3 POLYCYTHEMIA 51
4 ATYPICAL LYMPHOCYTOSIS 51
5 LEUKOCYTOSIS AND SPLENOMEGALY 52
6 PANCYTOPENIA 52
7 LEUKOCYTOSIS 52
8 LYMPHADENOPATHY 52

MODULE III & IV - HEMOSTASIS AND THROMBOSIS

1 A YOUNG BOY WITH SWOLLEN KNEE 68


2 A BLEEDING YOUNG WOMAN 68
3 FEVER AND COAGULOPATHY 69
4 A BLEEDER BABY BOY 69
5 BLEEDING GUMS AND BRUISING 69
6 A YOUNG PATIENT WITH SUDDEN SHORTNESS OF BREATH 70
7 OFTEN MISSED BLEEDING PROBLEM 70
8 POST OPERATIVE DVT 71
9 ALCOHOLIC AND COAGULOPATHY 71
10 FEVER POST SPLENECTOMY 71
11 BLEEDING WITH SEVER THROMBOCYTOPENIA 72

4
COURSE IN CLINICAL HAEMATOLOGY
investigations, and be able to interpret,
Aim integrate and correlate salient points in
clinical history, physical findings and
laboratory data with a view to making a
Extensive knowledge of common disorders as diagnosis of the type and cause of anemia in a
encountered in the practice of clinical hematology; patient
to enable the students to acquire requisite clinical - plan and prescribe a rational schedule of
skills for their diagnosis, and for planning and management, based on a sound knowledge of
carrying out appropriate interventions for their the mechanism of action, dose, duration and
prevention and management; to emphasize the side effects of appropriate drug therapy
emotional response and to understand the - identify patients with anemia requiring
consequences of patient’s sickness. blood component therapy, and to plan a
management schedule including the type,
amount, rate and the frequency of such
administration
Objectives - advise the patient and the family regarding
At the completion of the course in clinical measures to be adopted for prevention or
hematology, the student shall demonstrate: recurrence of anemia
­ Knowledge of cardinal manifestations of
hematological diseases
­ Comprehension of etiology, pathophysiology
pathogenesis, diagnosis, and principles of
Module II : White Blood Disorders
management of hematological diseases. At the completion of the study of disorders affecting
­ Understanding of the basic principles of white blood cells, the students shall be able to:
laboratory investigations of hematological - differentiate benign from malignant causes of
diseases, and the ability to interpret relevant leukocytosis
data to arrive at appropriate diagnosis of a - describe the clinical presentation, plan of
hematological disorder. investigations, and be able to interpret relevant
­ Ability to correlate relevant aspects of basic laboratory data, so as to reach likely diagnosis
sciences and molecular biology with the and prognosis in a patient who may present
clinical manifestations of hematological with leukocytosis and/or lymphadenopathy
diseases. - explain basic molecular abnormalities
­ Recognition of the impact of disease on underlying malignant transformation of
patient's social, emotional and professional haematopojetic cells, and resulting in
life. hematological malignancies
- plan and prescribe a rational schedule of
management, based on a sound knowledge of
Specific Learning Objectives the mechanism of action, dose, duration and
side effects of appropriate drug therapy
The course in clinical hematology consists of 4
units with specified learning objectives for each
unit.
Module III: Coagulation Disorders
Module I: Disorders of Red Blood Cells At the completion of the study of coagulation
At the completion of the study of the disorders of disorders, the students shall be able to:
red blood cells, the students shall be able to: - discuss common causes of bleeding disorders
- describe causes and pathophysiology of and their pathogenesis
common types of nutritional anemias. - describe the clinical presentation, plan of
- enumerate the common causes of investigations and interpret, integrate and
haemolytic anemias. correlate salient points in clinical history,
- discuss the pathophysiologic and molecular physical findings and laboratory data so as to
mechanisms that cause hemolysis, and to reach a correct diagnosis in a patient presenting
differentiate between intravascular and with a bleeding disorder
extravascular cause of hemolysis - describe the common mode of clinical
- describe the clinical presentation, plan of presentation and demonstrate ability to

5
interpret relevant laboratory data in patients
with following bleeding disorders
- Haemophilia Module IV: Thrombotic Disorders
- von Willebrand's disease
- Acquired coagulation deficiency secondary to At the completion of this unit of study, the
vitamin K deficiency, liver disease, chronic students shall be able to:
renal failure, and DIC. - discuss pathophysiologic mechanisms of
- discuss common causes of thrombocytopenia. thrombosis and describe molecular basis of
- plan and interpret salient points in clinical inherited thrombophilia
history, physical findings and relevant - enumerate common causes of acquired and
laboratory data to establish diagnosis of the inherited thrombotic disorders
type and cause of thrombocytopenia in a - demonstrate ability to correlate salient points
patient. in clinical history and physical findings and to
- Plan and administer rational therapy to a plan and interpret laboratory investigations so
patient with thrombocytopenia. as to make dignosis of a thromboembilic
- Describe the indications, schedule of disorder
administration and side effects of the use of - describe mechanism of action, dose,
fresh frozen plasma, cryoprecipitate and frequency, mode of administration, and
platelets concentrates in the management of adverse effects of heparin and coumadin
bleeding disorders anticoagulants

6
HAEMATOLOGY COURSE
TH
5 YEAR STUDENTS 2005-2006

FIRST WEEK
Date Time Topic Lecturer/Department
P.M.

Saturday 2 – 2.50 Lecture: Approach to Anemia Pathology


17/12/05
Medicine
3 – 3.50 Case Study: Microcytic Anemia in a Pathology
young woman

Sunday 2 – 2.50 Case Study: Macrocytic Anemia and Medicine


18/12/05 confusion Pathology

3 – 3.50 Lecture: Hemolytic Anemia Medicine


(Immune & Hereditary)

Monday 2 – 2.50 Case Study: Normocytic Anemia and Medicine


19/12/05 Reticulocytosis Pathology

3 – 3.50 Lecture: Hemoglobinopathy Pathology


(Thalassemia/SCA)

Tuesday 2 – 2.50 Communication Skills Community Med.


20/12/05
3 – 3.50 Case Study: A young man with recurrent Medicine
bony aches & jaundice Pathology

4 – 4.50 Case Study: Unexplained Microcytic Medicine


Anemia in a Kuwaiti patient Pathology

Wednesday 2 – 3.50 Practical (Anemia) Pathology


21/12/05

SECOND WEEK

Page 7 of 72
Saturday 2 – 2.50 Lecture: Acute Leukemia Medicine
24/12/05
Medicine
3 – 3.50 Lecture: Myeloproliferative Disorders
(CML/ET/PRV/MF)
Date Time Topic Lecturer/Department
P.M.

Sunday 2 – 2.50 Case Study: Medicine


25/12/05 Polycythemia in a smoker Pathology

3 – 3.50 Case Study: Medicine


Leukocytosis and Splenomegally Pathology

Monday 2 – 2.50 Lecture: Lymphoproliferative Medicine


26/12/05 Disorders
3 – 3.50 Medicine
Pathology
Case Study:
A young girl with neck lump

Tuesday 2 – 2.50 Communication Skills Community Medicine


27/12/05
3 – 3.50 Lecture: Blood Component Therapy Pathology

Wednesday 2 – 3.50 Practical (WBC disorder) Pathology


28/12/05

THIRD WEEK
Saturday 2 – 2.50 Lecture: Approach to patient with Medicine
Page 8 of 72
31/12/05 bleeding tendency

3 – 3.50 Lecture: Medicine


Thrombocytopenia/Hemophilia

Sunday 2 – 2.50 Case Study: Medicine


01/01/06 Woman with recurrent bleeding Pathology

3 – 3.50 Case Study:


Medicine
A bleeder child Pathology

Monday 2 – 3.50 MDS: Thrombosis Medicine


02/01/06 Pathology
Radiology

Tuesday 2 – 2.50 Case Study: Elderly man with Medicine


03/01/06 unexplained thrombocytosis Pathology

Medicine
3 – 3.50 A boy with painful knee
Pathology

Wednesday 2 – 3.50 Pre-exam Revision


04/01/06

FOURTH WEEK
Saturday HOLIDAY
07/01/06

Sunday 9.30 – HAEMATOLOGY ASSESSMENT Medicine


08/01/06 11.00 Pathology
AM

Monday HOLIDAY
09/01/06

Tuesday
10/01/06 HOLIDAY

Wednesday
11/01/06 HOLIDAY

Page 9 of 72
Approach to Anemia
Classification of Anemia
The classification of anemias is based on the size of the red blood cell and thus, divided into Microcytic
(hypochromic), Macrocytic and Normocytic (normochromic). Normograms for the anemias can be used to order the
appropriate tests for each group as follows:

Microcytic Anemia

F e r r it in

R educed N o rm a l In c re a s e d

I r o n d e f ic ie n c y H B e l e c t r o p h o r e s is A n e m i a o f C h r o n i c D is e a s e B o n e m a rro w
R i n g e d s id e r o b l a s t s

If HB electrophoresis is
Further investigations to normal then do alpha gene
find the cause are mapping
necessary

Hypochromia
Microcytic

Note: Anisocyosis: RDW


poikilocytes

Thalassemia Trait
Variables IDA
RBC count < 5 x1012/L > 5 x1012/L

MCV to HB level Proportional Disproportional

Mesner criteria
> 13 < 13
MCV/RBC count

Red cell
distribution width Elevated Normal
(RDW)
Anisocytosis
Morphology Poikilocytosis / Target cells
Elliptocytes

Page 10 of 72
Normocytic Anemia
It is important to have a retics count done in normocytic anemia. As a high retics count indicates a healthy
proliferating bone marrow. And this usually indicates hemolytic process or bleeding episode. It is important to
calculate the absolute retics count or retics production index. Please, look at tests listed above for the investigations of
Normocytic anemia.

R e t ic u lo c y t e s

In c re a s e d N o rm a l
D e c re a s e d

H e m o ly s is P o s t H e m o r r h a g ic B o n e M a rro w E x a m

I m m u n e v s . n o n - im m u n e A b n o rm a l N o rm a l

LDH/Bili/haptoglobin / Urine H y p o b l a s t ic RFT


hemosiderin I n f ilt ra t io n LFT
Coomb's test / Cold
D y s e r y t h r o p o i e t ic E n d o c r in e
agglutinins
F e r r it in
G-6PD / HAMS test / Osmotic
fragility
Polychromasia: expressed as high reitcs
count.

Fragmented RBC: schistocyte

Page 11 of 72
Macrocytic anemia
Here we need to differentiate between megaloblastic and non-megaloblastic anemias. In cases of megaloblastic
anemia we need to request schilling test in case of Vit.B12 deficiency. In case of Folate deficiency we need a detailed
dietary history, as poor dietary intake is usually responsible. Please look at the investigations listed below for
Macrocytic anemia.

B lo o d a n d M a r ro w M o r p h o lo g y

M e g a l o b l a s t ic N o n - M e g a l o b l a s t ic

C lin ic a l D a ta L iv e r d is e a s e
S e r u m V it a m in e s M yxed e m a
M D S

N o d e fic ie n c y F o la t e D e f ic ie n c y B 1 2 d e f ic ie n c y

C o n g e n it a l D ie t S c h ilin g t e s t
D ru g s

Dimorphic anemia

If things do not “compute’ i.e. the hematological picture dose not fit with scheme then consider dimorphic blood
pictures (two different combined types of anemia).

Page 12 of 72
HEMATOLOGY LABORATORY –
NORMAL VALUES
(Expressed in S.I. Units) Thrombin Clotting Time 9 – 11 sec
GENERAL HEMATOLOGY
Bleeding Time (IVY) 2 - 10 min
Test Male Female
WBC (x109/L) 4.0 – 10.0 4.0 – 10.0 FDP Assay 0 – 10 ug/L

RBC (x 109/L) 4.5 – 6.5 4.0 – 5.6 Fibrinogen Level 1.7 – 3.4 g/L

Hb (g/L) 135-180 115-160 Factor Assays 0.50 = 1.5 u/Ml

Het (L/L) 0.40 – 0.5 0.3 - 0.4 Antithrombin III 0.8 – 1.2 u/mL

MCV (fL) 79.0 – 97.0 79.0 – 97.0 D-dimer 0 – 0.5

MCH (pg) 27.0 – 32.0 27.0 – 32.0

MCHC (g/L) 320 – 360 320 – 360

MPV (fL) 7.5 – 11.1 7.5 – 11.1

RDW (um) 8.6 – 13.0 8.6 – 13.0

PLT (x109/L) 150 – 450 150 – 450

RETIC (x109/L) 10.0 – 75.0 10.0 – 75.0

ESR (mm/h) 0.0 – 7.0 0.0 – 15.0


(Wintrobe)

WBC Differential % Absolute (x 109/L)

NEUT 0 .40 - 0.75 2.5 – 7.5

Lymph 0.20 - 0.40 1.5 – 3.0

MONO 0.02 - 0.10 0.2 – 0.8

EOS 0.01 – 0.06 0.04 – 0.4

BASO < 0.01 0.00 – 0.1

COAGULATION
Prothrombin Time (P.T.) 9 – 13 sec

International Normalized Ratio


(INR) < 13

Partial Thromboplastin Time (P.T.T) 26 – 36 sec

Page 13 of 72
PRINCIPLES OF HEMATOLOGY TESTS
HEMOGLOBINOMETRY, CELL COUNTS
AND INDICES SIZE
Normocytic - normal size (7.5u diameter)
Accurate electronic cell counting has largely replaced Microcytic - 6u diameter
previous counting chamber methods in most Macrocytic - 9u diameter
laboratories. These procedures may, or may not, be Anisocytosis - variation in size
automated but either way the parameters shown below
may be measured or calculated. SHAPE
Normal
WBC 9
x 10 /L • Biconcave disc
RBC x 1012/L Poikilocytosis
Hb g/L • Variation in size
Hct L/L Oval (or elliptical cells)
MCV fL • Large numbers suggest hereditary elliptocytosis,
MCH pg but oval cells are also seen in many anemias
MCHC g/L (common in iron deficiency and thalassemia). Oval
RDW um macrocytes suggest megaloblastic anemia
Spherocytes
DIAGNOSTIC PATTERNS OF RBC INDICES • Due to membrane loss, producing round spheres
which are densely staining, and appear small.
1. Normal MCV, MCHC Large numbers suggest hereditary spherocytosis or
• Normocytic, normachromic. Consider aplastic immune hemolytic anemia (IgG type)
anemia, chronic disease, malignancies, renal Target cells
failure, endocrine hypofunction, hemolysis, or • Due to membrane excess. Suggest liver disease,
acute blood loss. Iron deficiency may also be thalassemia, hemoglobinopathy or post-
normocytic and normochromic initially. splenectomy state.
2. Low MCV ± low MCHC Fragmented cells
• Microcytic ± hypochromic. Consider iron • Many forms (schistocytes, helmet cells). Suggest
deficiency, chronic disease, thalassemia fragmentation hemolysis with prosthetic heart
and rarely sideroblastic anemia and lead valves, micro-angiopathic process (DIC, TTP,
poisoning. HUS)
3. Increased MCV Sickle cells
• Macrocytic. Consider Vitamin B-12 or folate • In homozygous form, mainly
deficiency, reticulocytosis, alcholism, liver disease Acanthocytes (spine cells)
and myelodysplasia • Liver disease, post splenectomy state
Spurr cells (echinocytes)
RED CELL MORPHOLOGY • Uremia, often artifact
Careful examination of a peripheral blood film is an
integral part of the initial laboratory investigation STAINING
confirming abnormalities detected by the Coulter Normachromic
Counter and seeking further clues to a specific
• Central pallor occupies one-third of RBC
diagnosis. At the same time, leukocytes and platelets
Hypochromic
should be assessed. Morphologic assessement
includes: • Decreased Hgb concentration (increased central
pallor).
Polychromasia
• Bluish-gray staining (indicates reticulocytes).

Page 14 of 72
INCLUSIONS • Qualitative – eg. Normoblastic vs
Basophilic stippling megaloblastic erythropoiesis. Presence or
• Ribosome precipitation; seen in many anemias, absence of iron stores. Ring sideroblasts
but helpful in diagnosis of thalassemia and lead 2. Assessment of marrow infiltrates
poisoning. Howell-Jolly bodies – nuclear • Leukemia, myeloma, metastatic carcinoma
remnants, usually seen in post-splenectomy. Closed bone marrow biopsy (Jamshidi needle)
Nucleated RBC's provides a better sample for assessment of cellularity
• Suggest extramedullary hematopoiesis (myeloid and is essential if the bone marrow examination is
metaplasia), active erythropoiesis (bleeding, being performed to detect marrow fibrosis, or
hemolysis) or metastatic cancer in bone marrow. neoplastic infiltration (carcinoma, lymphoma,
Heinz Bodies Hodgkin's disease)
• Require special supravital stain; represent
denatured Hgb and seen with oxidative hemolysis, SEDIMENTATION RATE
some enzymopathies and hemoglobinopathies. The popular "Sed rate" is one of the simplest, yet
Parasites poorly utilized test in laboratory medicine. The usual
• Eg. Malaria method used involves placing a known volume of
anticoagulated blood in a special tube of specified
length (100 mm Wintrobe, or 200 mm Westergren) and
DISTRIBUTION
allowing the red cells to sediment for one hour. In
Rouleaux
normal persons the red cells sediment only 7 – 15 mm
• Suggest immunoproliferative disease if extensive,
(Wintrobe) during this hour but this rate increases with
but lesser amounts common in many acute and many illness due to an increase in globulins eg
chronic diseases. increased gammaglobulins (in myeloma, liver disease
Autoagglutination or chronic infection) or acute phase reactants such as
• Aggregates of RBC's due to cold agglutinations. haptoglobin and fibrinogen (in any traumatic,
infectious, inflammatory or neoplastic illness)
RETICULOCYTE COUNT
Detects young erythrocytes (by precipitation The Sedimentation Rate has been used to differentiate
of residual RNA with special stain) and usually organic (elevated) from functional (normal) disease
provides a reliable source of erythropoiesis. and to diagnose or to follow progress of inflammatory
Reticulocytes last upto 48 hours in circulation and 1% and infectious processes. A normal Sedimentation Rate
erythrocytes are made each day; therefore, up to 2% does not rule out serious illness, and an elevated
reticulocytosis is considered normal (absolute 10 – 75 Sedimentation Rate is a non-specific finding.
x 109/L).
OTHER LABORATORY PROCEDURES
Reticulocytopenia A careful evaluation of some of all of the previous
• Suggests bone marrow depression, infiltration (eg basic laboratory tests may be sufficient for a diagnosis
leukemia) or lack of erythropoietin, iron, etc. but often they serve, in conjunction with clinical
Reticulocytosis assessment, to direct specific investigations. These
• Suggests hemolysis, blood loss, or response to investigations are described elsewhere. From this
treatment. point, careful selection of laboratory tests is essential.
If the reticulocyte count is reported as a percentage Widespread ordering of many expensive and
and the patient is anemic, it should be converted to an unnecessary investigations without regard to basic test
absolute count to assess effectiveness of reticulocyte results is wasteful and unrewarding.
response.
SPECIAL CONSIDERATIONS FOR
PEDIATRIC PATIENTS
BONE MARROW EXAMINATION
Most common indications for marrow aspirations The following differences between adult and pediatric
include patients require emphasis:
1. Assessement of cytopenias (anemia, neutropenia, 1. Level of Hemoglobin: At birth, the Hb is very
thrombocytopenia) high but shortly after birth, there is decrease in Hb
• Quantitative – eg. Hypocellular vs (as well as Hematocrit and RBC count). The
hypercellular lowest point is reached between 3 and 6 months
• Presence of absence of precursor cells (eg. with Hb as low as 100g/L being normal. By one
Megakaryocytes) year of age infants are often physiologically
slightly iron deficient. This is more marked in
Page 15 of 72
premature infants who require supplemental iron are small and not atypical as seen in viral
to avoid iron deficiency anemia. infections.

2. White Blood Count and Differential: Initially in 3. Lymph nodes: There is a marked lymphoid
neonates there is neutrophilic leukocytosis, but hyperplasia of the lymphoid system in children
within a week a lymphocyte predominance reaching a peak between 3 and 5 years. Tonsillar
develops and lasts until about five years of age enlargement is a common finding and enlargement
when the percentage of lymphocytes and of the cervical axillary and inguinal lymph nodes
granulocytes is equal. Later the child develops the is frequently found on routing examination.
usual adult picture of neutrophil predominance.
Pertusis causes a striking lymphocytosis with
white counts up to 100 x 109/L. The lymphocytes

Page 16 of 72
Basophilic stippling Burr Cells Elliptocytes

Heinz Bodies Helmet cells Hemoglobin C crystals

Howell-Jolly bodies Hypochromia Macroovalocytes

Malaria-Plasmodium Malaria- Plasmodium Megakaryoblast and


Falcipuram Vivax promegakaryocyte

Megalocytes-1 Megalocytes-2 Microcytosis

Pappenheimer bodies- iron Pappenheimer bodies-


stain Wright stain Poikilocytes

Page 17 of 72
Polychromatophils Rouleaux Sickle Cell

Spherocytes Target Cells Teardrop cells

Page 18 of 72
MODULE I
DISORDERS OF RED BLOOD CELLS
OBJECTIVES

• Describe causes and pathophysiology of common types of nutritional anemias.

• Enumerate the common causes of haemolytic anemias.

• Discuss the pathophysiologic and molecular mechanisms that cause hemolysis,


and to differentiate between intravascular and extravascular cause of
hemolysis.

• Describe the clinical presentation, plan ofinvestigations, and be able to


interpret, integrate and correlate salient points in clinical history, physical
findings and laboratory data with a view to making a diagnosis of the type and
cause of anemia in a patient.

• Plan and prescribe a rational schedule of management, based on a sound


knowledge of the mechanism of action, dose, duration and side effects of
appropriate drug therapy.

• Identify patients with anemia requiring blood component therapy, and to plan
a management schedule including the type, amount, rate and the frequency of
such administration.

• Advise the patient and the family regarding measures to be adopted for
prevention or recurrence of anemia.

Page 19 of 72
NUTRITIONAL ANEMIA:
IRON ABSORPTION AND 4. Intravascular Hemolysis
METABOLISM Paroxysmal nocturnal hemoglobinuria (an
exceedingly rare disease) and much more
Iron is absorbed largely in the duodenum and upper
commonly, intravascular hemolysis due to
jejunum. The absorption of ferric iron, but not ferrous
erythrocyte trauma from a cardiac valvular
or hemoglobin iron, is facilitated by hydrochloric acid,
prosthesis may both result in an iron deficiency
and impaired after gastrectomy. The rate of iron
Intravascular hemolysis leads to trapping of
absorption is regulated by physiological needs, and is
hemoglobin in renal tubular cells followed by
increased in iron deficiency and other anemias. Human
formation of hemosiderin which is lost in
control their total body iron by absorption and not iron
desquamated cells. Hence, hemosiderinuria occurs
excretion.
with a loss of as much as 15 mg of iron per day.
The average North American diet contains about
10 - 15 mg of iron daily. Total daily iron loss in men is
about 1 mg (0.5 mg in feces, 0.1 mg in sweat). The CLINICAL FEATURES OF IRON
average daily iron loss in women during reproductive DEFICIENCY
life is about 2 mg. The iron deficient patient may be completely a
symptomatic, even in the presence of marked anemia if
IRON DEFICIENCY this has developed gradually. There is good evidence
that the anemia itself may not be responsible for the
Iron depletion without anemia is very common;
symptoms, muscle, etc. Symptoms frequently reported
some degree of iron deficiency is present in
include: fatigue, tinnitus, palpations, weakness and
approximately 10% of most populations. Mild
lightheadedness. Sore tongue or sore mouth sometimes
anemia, due to iron deficiency, is frequently not
occurs and there may rarely be severe stomatitis. There
hypochromia and there are other causes of
may be some loss of papillae from the tongue and
hypochromic anemia other than iron deficiency; hence
fissures at the corners of the mouth. Mild
hypochromic anemia and iron deficiency anemia are
splenomegaly occurs rarely. Koilonychias (spoon
not synonymous.
nails), a condition in which nails are flattened or even
ETIOLOGY OF IRON DEFICIENCY concave and are brittle, occurs uncommonly but is of
SPECIFIC CAUSES considerable diagnostic help when it occurs.
1. Menstruation
The following points are important to assess the LABORATORY INVESTIGATION
severity of menstrual bleeding:
1. Examination of the Stained Blood Film and
A. The number of pads used. Greater than 12
pads is considered abnormal. Red Cell Studies.
B. If double pads are used, does blood soak The classical appearance of red cells in iron
through? deficiency is that of hypochromia and microcytosis.
C. Are there large clots? These features are often helpful in making a
D. How many days do the periods last; more than diagnosis but two problems prevent complete
five days being suggestive of increased reliance on it. One is that there are causes such as
bleeding. "anemia of chronic disease", thalassemia and
E. Pregnancy and Lactation: sideroblastic anemia for erythrocyte hypochromia
other than iron deficiency. The second problem is
2. Impaired Absorption of Iron
that the anemia of iron deficiency may be
This is rarely a cause of iron deficiency except in
normochromic and normocytic until quite severe.
the presence of sprue or as the result
gastrointestinal surgery. Most patients who have 2. Serum Iron and Iron Binding Capacity
had total gastrectomy and up to 50% of patients Determination (these studies may be limited
with sub-total gastrectomy develop iron values)
deficiency, although this may require several years Normal Range
after surgery. Total Iron Binding Capacity 45 - 81
umol/L (TIBC)
3. Gastrointestinal Bleeding
Unsaturated Iron Binding Capacity 27 - 54
Determining the source of gastrointestinal
umol/L (UIBC)
bleeding is a challenge. Radiologic examinations
and endoscopy are the chief aids to the diagnosis.
Serum Iron
With respect to gastric bleeding, corrosive injury *Percent saturation = serum iron x 100%
from several drugs (e.g. Aspirin, phenylbutazone, Total iron binding capacity
Indomethacin) occurs with considerable frequency. *Values of less than 15% are suggestive of
Page 20 of 72
iron deficiency. c. An iron-deficient patient with normal
3. Serum Ferritin gastrointestinal absorptive function will respond
The level of ferritin in the plasma appears to be a to any oral iron preparation give in an adequate
reliable indicator of the size of body iron stores in dosage for an adequate period of time.
most situations. The normal range is d. Patient tolerance is largely related to dosage.
approximately (adult male: 15-350 ug/L; adult e. Ferrous sulphate is much cheaper than any other
female: 15-200 ug/L). Values below the normal iron preparation.
range indicate iron deficiency and correlate well
with absent marrow iron. With chronic 2. Oral Iron Preparations
inflammatory or malignant disease, ferritin tends In a daily dose of 0.9 grams given as 300 mg
to be elevated or at least normal, even when iron t.i.d., p.o., ferrous sulphate results in rapid
stores are absent. Therefore, a low serum ferritin is correction of iron deficiency anemia after a latent
diagnostic of iron deficiency, whereas a normal period of approximately 14 days. Three hundred
level in a patient with chronic inflammation, may (300) mg of ferrous sulphate t.i.d. provides 180 mg
not actually reflect the iron stores. of elemental iron per day, of which about 10% is
4. Bone Marrow absorbed by the normal individual and up to 25 or
Iron stores are usually evaluated following even 40% in the iron-deficient individual.
Prussian blue staining of the bone marrow smear. Replenishment of iron stores requires at least four
This is a highly reliable method of assessing iron months of treatment after the return of the
deficiency, since this store is exhausted before hemoglobin concentration to normal. Thus, in the
anemia occurs. absence of continuing blood loss, oral iron therapy
5. Examination of Stools for Blood should be continued for at least six months.
Occult bleeding causing iron deficiency is usually Continued blood loss requires more chronic
of too small a volume to cause blackening of therapy.
the stool. Testing for occult bleeding may be done
with the use of benzidine, Guaiac or orthotolidine.
3. Parenteral Iron Therapy.
The indications for parenteral iron therapy are
very limited since oral therapy is
TREATMENT OF IRON DEFICIENCY Generally well tolerated, effective and inexpensive.
1. Principles of Treatment: (Determine the
cause)
a. Iron deficiency is the only disorder that
responds to iron administration.
b. A search for chronic blood loss often must
accompany treatment.

Page 21 of 72
MEGALOBLASTIC ANEMIA
Megaloblastic anemia is a term used for an anemia VITAMIN B12 DEFICIENCY
with a macrocytic blood film and megaloblastic
NUTRITIONAL REQUIREMENTS
erythroid maturation in the bone marrow. These
Vitamin B12 is required in small amounts (1 - 2 ug)
changes are due to defective DNA synthesis, most
daily and since adequate B12 is found in all foods of
often secondary to deficiency of vitamin B12, or folic
animal origin it is also impossible for anyone other
acid. Other organs, such as G.I. tract and CNS may be
than a strict vegetarian to become B12 deficient on a
affected.
dietary basis. Vitamin B12 is not present in vegetables
The common findings are:
and fruit. Body stores are 2 - 3 mg enough to last 3 - 4
years.
PERIPHERAL BLOOD
Pancyopenia: A moderate reduction in leukocytes
and platelets is usual, and anemia may vary from mild ABSORPTION
to severe. Normal B12 is absorbed selectively in the ileum by the
Blood film: The erythrocytes vary greatly in size intrinsic factor mechanism. Intrinsic factor is a
and shape, but macrocytes predominate and oval glycoprotein, secreted by gastric parietal cells. It
macrocytes may be seen. The neutrophils have quickly binds with vitamin B12 released from food in
hypersegmented nuclei (6 or more lobes). the stomach, and transports the B12 to specific sites of
Polychromasia is not conspicuous (and reticulocyte attachment on the brush border of the ileal mucosa.
count is not increased). The B12 is absorbed, after several hours delay in the
ileal mucosa, and is carried in the blood stream
attached to proteins (transcoballamins).

CAUSES OF VITAMIN B12 DEFICIENCY


Nutritional - vegans
MALABSORPTION
In comparison to the Gastric causes - pernicious anemia (P.A.)
lymph, many RBC are Hypersegmented ­ gastrectomy
large and oval Neutrophil Intestinal causes - Blind loop syndrome
­ sprue, ileal resection
­ Crohn's disease
Bone Marrow: erythroid hyperplasia, and ­ Fish tape
megaloblastic maturation as shown below
. PERNICIOUS ANEMIA
Giant metamyelocyte Dysmegakaryopoiesis Adult P.A. occurs in both men and women, usually
Erythroblast over the age of forty, and is characterized in the typical
case by triad of:
a) Megaloblastic anemia:
b) Glossitis: recurrent sore tongue and mouth, with
progressive atrophy of papillae, leading to a
smooth, red tongue.
OTHER LABORATORY TESTS c) CNS changes: peripheral neuritis is common, and
Due to the breakdown of erythroid cells, mainly in subacute combined degeneration is unique to B12
the bone marrow (intramedullary hemolysis) the deficiency (it does not occur with folate deficiency).
following abnormalities may be seen, although there is
no need for them to be ordered: AUTOIMMUNE ASPECTS:
1. Serum bilirubin - mild increase, mainly indirect It has been suggested that P.A. is an "autoimmune
fraction disease". The B12 malabsorption is secondary to lack of
2. Lactic dehydrogenase (LDH) - may be greatly intrinsic factor production, due to gastric atrophy. The
increase; LDH1 predominates gastric mucosa is invaded by lymphocytes and plasma
3. Decreased haptoglobin, and increased cells, and there is a high incidence of parietal cell
methemalbumin. antibody (95%) and intrinsic factors antibody (30 -
60%).

Page 22 of 72
LABORATORY INVESTIGATION
The specific laboratory tests are: 2. Gastric Juice
1. Serum B12 All adult patients with P.A. have achlorhydria but is
Vitamin B12 is measured in the serum by not common practice to measure this.
radioimmunoassay. Serum B12 should be measured
3. Defective B12 Absorption
usually along with folate, before any treatment is
The Schilling test may be performed if doubt exists as
given, or a Schilling test performed. A low serum B12
to B12 absorption.
and normal serum folate is virtually diagnostic of B12
deficiency; further tests are necessary to determine the
cause. A slight decrease in serum B12 may occur with TREATMENT
severe folate deficiency. Serum B12 may be reduced in In pernicious anemia, 1000 ugm cyano-cobalamin
patients on birth control pills. intramuscularly each month will sustain a normal
blood picture.

FOLIC ACID DEFICIENCY


REQUIREMENTS AND DIETARY SOURCE activity and therefore conversion to active co-
Man is unable to synthesize his estimated daily enzyme forms.
requirement of about 50 ug per day (infants 25-50 ug 4. Alcohol
per day). Meats, particularly liver, and green The folate deficiency of alcoholics is complex and
vegetables are good dietary sources of folic acid. probably involves dietary deficiency as well as
Boiling vegetables in excess water will efficiently impaired absorption and perhaps also decreased
extract much of the folate content. Requirements are hepatic storage and altered utilization.
increased in pregnancy, hemolytic anemia, and 5. Pregnancy
hyperthyroidism. Megaloblastic anemia of pregnancy usually
responds to folic acid and usually
ABSORPTION AND FUNCTION OF FOLIC laboratory evidence of folate depletion is obtained.
ACID Requirements for the pregnant woman are not well
Folic acid is absorbed in the duodenum and jejunum established, but are increased and pregnant women
by an active transport mechanism. Folic acid should receive supplemental folic acid (as well as
metabolites act as coenzymes in the metabolism of iron) to prevent deficiencies.
both DNA and RNA. There is a complex
interrelationship between vitamin B12 and folic acid LABORATORY INVESTIGAIONS OF
metabolism and function such that DNA synthesis is FOLATE DEFICIENCY
impaired with a deficiency of either vitamin and Normal Values
pharmacological doses of the opposite vitamin will Serum Folate 6.8 - 18.4 nmol/L
partly correct the impaired DNA synthesis, Red Cell Folate 363 nmol/L
megaloblastosis and anemia.
Low serum and red cell Folate with normal or slightly
CAUSES OF FOLIC ACID DEFICIENCY reduced B12 levels is diagnostic of Folate deficiency.
1. Dietary Deficiency As with B12 deficiency, the megaloblastosis in the
Requires approximately four months to produce marrow is rapidly corrected, and reticulocytosis is
anemia in the healthy subject. prompt (3 - 7 days).
2. Intestinal Disease
Especially gluten enteropathy (coeliac disease) and TREATMENT OF FOLIC ACID
tropical sprue. DEFICIENCY
3. Drugs In addition to removing causative factors where
Commonly dilantin, primidone, and barbiturates. possible, the administration of folic acid orally is
Oral contraceptive agents also appear occasionally usually required.
to cause malabsorption. The folic acid antagonist
methotrexate inhibits dihydrofolate reductase

Page 23 of 72
ALCOHOL AND HEMATOPOIESIS
Chronic and excessive alcohol ingestion causes many are the morphological clue to this type of
hematological complications by direct toxic effects, by hemolysis.
contributing to nutritional deficiency or by
complicating liver dysfunction as follows: 4. Sideroblastic Anemia
Direct toxic effect of alcohol on hemesynthesis
1. Macrocytic Anemia causes ring sideroblasts, ineffective erythropoiesis,
Usually due to folic acid deficiency on a and a hypochromic anemia.
nutritional basis (megaloblastic anemia) but even
in the absence of folate deficiency, alcohol and 5. Hypersplenism
liver disease can cause macrocytic anemia. Due to cirrhosis and portal hypertension causing
congestive splenomegaly. Pancytopenia is usually
2. Iron Deficiency Anemia present.
Usually due to chronic G.I. bleeding from
esophageal varices or gastritis Thrombocytopenia, often severe, may be caused
by a direct toxic effect of alcohol, even in the
3. Hemolytic Anemia absence of folate deficiency.
Usually a complication of severe alcoholism with
liver disease and often hyperlipidemia. Spur cells

APLASTIC ANEMIA
Aplastic anemia is an uncommon, but often serious, 2. Anti-inflammatory drugs – phenylbutazone,
blood disorder characterized by pancytopenia due to indomethacin, gold, etc
the decreased functional capacity of a hypoplastic 3. Diuretics – thiazides
(fatty) bone marrow. Even in severe cases of aplastic 4. Anticonvulsants – dilantin
anemia there may be small residual islands of
functioning bone marrow which may cause confusion Chloramphenicol rarely causes a severe irreversible
in diagnosis, and which lead to a concept that the aplastic anemia (1 in 30,000 cases treated) which
micro-environment (the soil) rather than the stem cell cannot be predicted, but frequently causes a mild
(the seed) might be abnormal in these patients. reversible pancytopenia (Idiosyncratic reaction).
However, the success of bone marrow transplantation Phenylbutazone is now the commonest cause, since the
strongly supports the concept that deficient or use to chloramphenicol has decreased.
defective stem cells are responsible. The aplastic anemia associated with viral hepatitis is
severe and often fatal. Immunologic causes of some
CLASSIFICATION cases of aplastic anemia have been postulated but not
proven
IDIOPATHIC: - Heriditary (Fanconi's
anemia) CLINICAL AND LABORATORY FEATURES
- Acquired Pallor, bleeding and bruising are common and
SECONDARY - Chemical and Drugs infections are frequent. Lymphadenopathy and
- Radiation hepatosplenomegaly are characteristically absent. The
- Infection (viral hepatitis) anemia may be severe, usually normocytic but
- Immunologic (?autoimmune) sometimes slightly macrocytic. Granulocytopenia is
regularly seen, and lymphopenia is variable.
The cause of about 50% cases cannot be determined Thrombocytopenia is usually marked, and
and these are termed idiopathic. Fanconi's anemia is an megathrombocytes are absent.
extremely rare hereditary aplastic anemia, often
associated with short stature, skeletal abnormalities, MANAGEMENT
hypoplastic kidneys, etc. The role of red cell, granulocyte, and platelets
Exposure to chemicals containing benzene is transfusions are discussed with the management of
particularly hazadarous. Drugs which most frequently leukemias. When aplastic anemia is severe
are associated include: (granulocyte count 0.5 x 10 9/L, platelet count 20 x 10
9/L) and not reversible, the prognosis is very grave
1. Antibiotics – chloraphenicol, sulfas, streptomycin (85% mortality in one year, with median survival of 3

Page 24 of 72
months). If an HLA-compatible sibling is available as high dose androgen therapy may cause improvement in
a bone marrow donor, then prompt bone marrow anemia but not leucopenia nor thrombocytopenia.
transplantation is the treatment of choice (not that an Steroids are often given a trial of 2 -4 weeks but
HLA-compatible sibling is only available in 1/4 to 1/3 should then be discontinued if not obviously helpful.
of cases. Splenectomy has no definite role to play in these cases.
For milder cases, or those unsuitable for The status of immunosuppressive treatment
marrow transplantation Antithymocyte globulin can (Cyclosporin) is investigational but helpful in some
cause a significant improvement in a number of cases. cases.

THE HEMOLYTIC ANEMIAS


Increased RBC destruction is the common factor in all II EXTRINSIC DEFECTS
hemolytic anemias. Hemolysis is detected by: (EXTRACORPUSCULAR)
1. Evidence of erythrocyte destruction.
2. Bone marrow compensation 1. Autoimmune hemolytic anemia (Coombs positive)
3. Decreased erythrocyte survival a. Warm antibody (IgG)- idiopathic, secondary
(lupus, lymphoma, Aldomet, etc)
EVIDENCE OF ERYTHROCYTE b. Cold antibody (IgM) – idiopathic (cold
DESTRUCTION hemagglutinin disease), secondary
a) Hemoglobin, hematocrit – both decreased unless (mycoplasma, infectious mononucleosis)
hemolysis is "compensated" (normal bone marrow 2. Alloimmune hemolytic anemias – Hemolytic
can increase RBC production at least 6 to 8 times) disease of newborn, post transfusion
b) Hyperbilirubinemia – increased serum bilirubin, 3. Drug induced hemolytic anemia
mainly indirect (unconjugated type), and increased a. Chemical effect in absence of G6PD
urobilinogen in urine. deficiency, or with G6PD deficiency
c) * Increased plasma hemoglobin 4. Mechanical hemolytic anemia
* Decreased serum haptoglobin a. Micro-angiopathic – hemolytic – uremic
* Hemoglobinuria syndrome, TTP, DIC, etc
* Hemosideriuria b. Cardiac fragmentation hemolysis
* Mainly intravascular hemolysis 5. Secondary hemolytic anemia – infectious, liver or
renal disease, etc.
BONE MARROW COMPENSATION
a) Reticulocytosis – seen as polychromasia on routine I INTRINSIC DEFECTS
blood films, but can be specifically demonstrated
on reticulocyte count. A. HEREDITARY HEMOLYTIC ANEMIAS
b) Erythroid hyperplasia of bone marrow
1. MEMBRANE DEFECTS:
DECREASED ERYTHROCYTE SURVIVAL Hereditary Spherocytosis is an uncommon
Seldom necessary Cr51 most often used as radioactive form of hemolytic anemia (incidence of 1 in 5,000
label for patient's own erythrocytes. Requires 10 to 14 population) with an autosomal dominant
days. inheritance. Spherocytes are usually evident on
examination a routine blood film. If not obvious,
CLASSIFICATION then an osmotic fragility test should be ordered to
I INTRINSIC DEFECTS detect the osmotically fragile pre-spherocyte cells.
(INTRACORPUSCULAR) Reticulocytosis and splenomegaly are usually
prominent. Splenectomy is the definitive treatment
A. HEREDITARY after which hemolysis ceases, although
1. Membrane defect – HS, HE spherocytes persist in the blood.
2. Hemoglobinopathies Hereditary Elliptocytosis is also called
a. Quantitative – Thalassemias hereditary ovalocytosis and is relatively common
b. Qualitative – Hb S, C, S-C etc (1 in 2,000 population) with an autosomal
3. Enzymopathies – G6PD, PK, etc dominant inheritance. Most cases have little or no
hemolysis but 10% or less have significant
B. ACQUIRED hemolytic anemia requiring splenectomy.
1. Paroxymal Nocturnal Hemoglobinuria (PNH)
Page 25 of 72
2. HEMOGLOBINOPATHIES: Diagnosis of enzymopathies requires demonstration
Thalassemia is a complex group of anemias of the specific enzyme deficiency by complex
usually classified with the hemolytic anemias laboratory procedures.
because hemolysis is marked in the severe
homozygous forms, but the primary defect is a B. ACQUIRED HEMOLYTIC ANEMIAS
quantitative reduction in synthesis of globin 1. PAROXYSMAL NOCTURNAL
chains. Beta chain synthesis is decreased in beta HEMOGLOBINURA (PNH);
thalassemia which is mainly seen in individuals of Very rare hemolytic disorder. Acquired increased
Mediterranean background. sensitivity of the RBC membrane to complement,
The heterozygous form (beta thalassemia minor) causing intravascular hemolysis and hemoglobinura.
is fairly common is characterized by extreme Diagnostic test is Ham's acidified serum test. PNH
microcytosis, and hypochromasia but little or no may be associated with aplastic anemia or
anemia. Diagnosis is confirmed by demonstrating myelofibrosis.
an increased Hb A2, and no treatment is required Treatment: supportive blood transfusions.
(although laboratory findings resemble iron
deficiency, iron stores are usually normal or II EXTRINSIC DEFECTS
increased and iron therapy is contradicted) The (EXTRA CORPUSCULAR)
homozygous form (beta thalassemia major) causes
severe hemolysis in childhood with splenomegaly, 1. AUTOIMMUNE HEMOLYTIC ANEMIA
and iron overload usually leading to death in (AIHA):
adolescence unless vigorous transfusion and iron- Requires the presence of:
chelation therapy is employed. a. Hemolytic anemia
Alpha chain synthesis is decreased in b. Antibody directed against an intrinsic antigen
alpha thalassemia which is commoner in on the patient's own erythrocytes; find
Orientals; the inheritance is more variable antibody or complement on the surface of the
resulting in clinical disorders ranging from a very patient's erythrocytes (direct antiglobulin or
mild anemia, through moderate hemolytic anemia Coombs test), and often antibodies in the
(Hb H disease), to a very severe form causing patient's serum (indirect antiglobulin or
death in utero. Coombs test)
Sickle cell Anemia- Sickle cell trait is common IgG antibodies lead to destruction of RBC's in
(10% of American Blacks) but is usually the R-E system of spleen and liver mainly. IgM
asymptomatic and detected only by the presence of antibodies usually fix complement and produce
abnormal Hb electrophoresis (SA) or a positive intravascular hemolysis.
sickle cell preparation. If the heterozygous sickle
cell trait is combined with the gene for Hb C, then a. WARM ANTIBODY TYPE (IGG):
a more severe disorder (Hb S-C disease) occurs This is the commonest type, usually chronic,
with hemolysis, splenomegaly, and hemolytic and often (50%) secondary to other diseases
crises. Sickle cell anemia (or disease) is the (lupus, lymphoma, chronic lymphocytic leukemia).
homozygous form (Hb SS) characterized by a Splenomegaly and lymphadenopathy suggest
severe hemolytic anemia beginning in early secondary type. Drug-induced hemolytic anemias
childhood, causing splenic infarcts occur by these mechanisms:
(autosplenectomy), bone infarts, hemolytic or i. Innocent bystander type- quinidine
aplastic crises and often death in young adult life. ii. Hapten type – penicillin (massive
Treatment is unsatisfactory (supportive care with doses)
transfusions, fluid therapy and analgesia). iii. Autoimmune type – Almodet (alpha
methyl DOPA); a positive direct
3. ENZYMOPATHIES: antiglobulin (Coombs) test occurs in
Hereditary deficiencies of G6PD or Pyruvate one third of patients on drug for
kinase (PK) are commonest. The gene for G6PD several months, but hemolysis is very
deficiency is sex-linked and many isoenzyme rare.
types exist. Most lead to acute hemolysis on Diagnosis is suggested by hemolysis,
exposure to oxidant drugs, or possible FAVA spherocytosis and polychromasia, and confirmed
beans, while others cause chronic hemolysis. by a positive direct antiglobulin (Coombs) test.
Blacks and Mediterranean races are most Treatment includes:
commonly affected. Management involves 1. Steroids – up to 60 – 80 mg Prednisone daily
avoidances of oxidant drugs. PK deficiency causes with gradual tapering depending upon
chronic hemolysis and splenomegaly; splenectomy response.
is usually helpful.

Page 26 of 72
2. Splenectomy – 50 % response; reserve for thrombotic thrombocytopenic purpura
cases unresponsive to steroids, or requiring (TTP) septicemia, DIC and carcinomatosis
large maintenance steroid dosage. may cause RBC fragmentation with or
3. Blood transfusion – difficult to crossmatch; without thrombocytopenia
use least incompatible blood and transfused ii. Cardiac Hemolytic Anemia – Due to
only if absolutely necessary. prosthetic aortic valve usually;
4. Immunosuppressive therapy (Imuran, occasionally with Teflon patch, or
Cyclophosphamide) – for cases unresponsive calcified valve. Iron deficiency is
to steroids and splenectomy. commonly associated because the chronic
5. Folic Acid (5 mg daily) – this is given to all intravascular hemolysis leads to chronic
chronic hemolytic anemias as prophylaxis for urinary iron (hemosiderin) loss.
folic acid deficiency due to increased folate
utilization 5. SECONDARY HEMOLYTIC ANEMIA:
A variety of infections (eg. Clostridia, malaria)
b. COLD ANTIBODY TYPE (IGM): may cause erythrocyte damage and hemolysis.
These are uncommon and Hepatic and renal disease result in mild chronic
characterized by RBC agglutination and hemolysis usually, but defective erythropoiesis
hemolysis when exposed to cold temperature. from chronic disease, iron deficiency may also
They may be: play a role
i. Idiopathic – chronic cold agglutinin
disease (CAD); relatively benign
hemolytic anemia in older persons, with
Raynaud's syndrome and hemoglobinuria
ii. Secondary – with lymphoma, or infectious
(mycoplasma, infectious mononucleosis)
Diagnosis is suggested by
autoagglutination on blood film and hemolysis; it
is confirmed by demonstrating increased cold
agglutinatinins (anti-I or anti-i)
Treatment involves avoiding cold
temperature and possible blood transfusion.
Chronic cases may need chemotherapy
(Chlorambucil). Steroids and splenectomy are not
indicated.

2. ALLOIMMUNE HEMOLYTIC ANEMIAS:


These are due to passage of maternal antibodies
across placenta to damage fetal RBC's (hemolytic
disease of new born) or to blood transfusion with
antibodies present acting against foreign RBC's.
Delayed hemolytic transfusion reactions of varying
severity may occur.

3. DRUG-INDUCED HEMOLYTIC ANEMIA:


Chemical Effect – Many oxidant drugs
(Phenacetin, Salazopyrine, etc) in high dosage
cause hemolysis in normals, but G6PD deficient
patients are particular susceptible to many drugs in
small amounts. The end result is methemoglobin,
sulfhemoglobin and Heinz body formation with
resultant RBC fragmentation.

4. MECHANICAL HEMOLYTIC ANEMIA:


The trauma to RBC membrane produces
fragmentation (schistocytes, helmet cells, burr
cells, spherocytes, etc). May be
i. Micro-angiopathic Hemolytic Anemia. –
Toxemia, abruption placenta, malignant
hypertension, hemolytic-uremic syndrome,
Page 27 of 72
BLOOD TRANSFUSION PRACTICE
DONOR AND PATIENT SELECTION Each hospital will have guidelines for appropriate
Donors should be volunteers, and are carefully procedure (cross match or group and reserve) for
screened for history of infectious disease and each type of operation.
medications. However, no transfusion can be
completely free of risk because: 2. BLOOD COMPONENT THERAPY
1. Present screening procedures do not eliminate a. Whole Blood – stored up to 5 weeks in CPD
hepatitis transmission (especially non-A, non-B anticoagulant. Used for treatment of acute blood
hepatitis). HIV screening of all blood products loss and replacement of red cell mass and volume)
occurs now. rarely available; blood components are used to
2. Some patients may react to non-RBC components provide the equivalent of whole blood in those
– e.g. allergic and febrile reactions. situations
3. Auto-antibodies produced to RBC antigens (other b. Packed Red Cells – storage and cell survival same
than ABO and Rh) may limit future transfusion as whole blood. Preferred for most anemias
therapy. requiring transfusion.
c. Plasma Products –fresh frozen plasma, stored
In spite of this, most transfusions (95%) produce plasma, albumin, cryoprecipitate and factor
no adverse effects, and serious reactions are rare. concentrates.
Patient selection depends on clinical judgement but the d. Platelet Concentrate
following guidelines might be used: e. Gamma Globulin and Specific Antisera –
Intravenous and intramascular gamma globulin
1. Degree and chronicity of anemia: Patients with preparations are available, as are specific antisera
chronic anemia of moderate degree (Hb 60 – 80 (anti-hepatitis B, CMV, Zoster)
g/L) often maintain reasonable activity, and the
need for transfusion should be correlated with 3. TRANSFUSION PROCEDURE
symptomatology (eg severe fatigue, aggravation of a. Rate of Transfusion. Depends on clinical situation.
angina, heart failure etc) One unit of blood may be infused in 15 minutes in
2. Active or potential bleeding: It is generally rapidly bleeding patients. Packed cell transfusions
accepted that Hb should be kept about 100 g/L in for anemia usually take about 2 hours (longer than
bleeding patients and in patients who undergo 4 hours should be avoided due to risk of
surgery. contamination)
3. Possible response of hemantinics. Usually better b. Amount Transfused. Depends on clinical situation.
to investigate anemia, and treat specifically (eg Single unit transfusions are generally avoided. The
iron, B12) if possible. patient's Hb should increase approximately 10 g/L
per unit of packed cells transfused, in average size
adult.
PRINCIPLES OF BLOOD TRANSFUSION
c. Diluents. Normal saline is the recommended
1. TYPE OF CROSSMATCH
diluent for blood products. A small amount (50
a. Group and Reserve Serum – no crossmatch;
mL) will correct the increased viscosity of packed
consists of blood group and antibody screen;
red cells (about 0.70). Plasma and 5% albumin are
serum is reserved for cross-matching later, if
possible alternatives. Glucose solutions cause RBC
needed. Some institutions do not crossmatch if the
agglutination and hemolysis so must not be used.
antibody screen is negative
Ringer's solution contains calcium which can
b. Routine Crossmatch- complete; takes atleast one
overcome the citrate anticoagulant and cause clots
hour.
to form in the tubing; so red cells must not be
c. Urgent or Emergency Crossmatch – shorter
diluted with Ringer's solution.
incubation (about 30 minutes) to provide blood for
urgent clinical situations.
ADVERSE EFFECTS OF BLOOD
d. Uncrossmatched Blood – give ABO and Rh type TRANSFUSIONS
same as patient (rather than Rh negative). Only
When any significant reaction, which cannot be
given in most extreme emergencies, unless patient
explained or treated, occurs during a transfusion, it is
has been previously tested (group and reserve
advisable to terminate the transfusion and investigate
serum) in which the case the need for
for hemolytic transfusion reaction. However, most
crossmatching is much less
reactions are not hemolytic.
Types of reactions include:
Page 28 of 72
1. HEMOLYTIC REACTIONS: these products are utilized to their best possible
a. Immediate. Chills, fever, dyspnea, bone pain, advantage. The following guidelines are currently
hypotension, red urine, renal failure and bleeding employed:
(D.I.C) may all occur. Terminate transfusion and
investigate by sending unused blood, venous 1. FRESH FROZEN PLASMA (FFP):
blood specimen and urine specimen to Blood Storage: 20oC for 1 year
Bank (for repeat crossmatch, direct Coomb's test Contents: Must be frozen within 12 hours of
and test for hemoglobinura) May need supportive blood donation so that it will contain all
treatment for shock, bleeding or renal failure. coagulation factors. No platelets are present, and it
b. Delayed. Fever, jaundice, or recurrent anemia contains allo-antibodies as in stored plasma.
occurring 3 days or more after transfusion. Use: Replacement of coagulation factors
Investigate for hemolysis, including Coomb's which cannot be provided by other products (i.e.
test. This may reflect an amnestic antibody for factor VIII alone, in hemophilia, factor VIII
response to previous antigen exposure. concentrate would be used) Its main use is for
treatment of multiple coagulation deficiencies
2. FEBRILE REACTIONS: occurring after massive transfusion (10 or more
Chills, fever and occasionally hypoxia usually in units) or with D.I.C. Time is required for thawing
multi-transfused or multiparous patients with (up to 30 minutes) so stored plasma 5% albumin is
WBC antibodies. Treat with antipyretic, and try to preferred for immediate replacement in severe
prevent in future with leukocyte poor blood or hemorrhage. It is evident that excessive use of this
washed packed cells. There are several methods to component will mean less cryoprecipitate, or
produce leukocyte poor blood, leukocyte factor VIII concentrate, can be made, thereby
centrifugation in line filters, washed packed cells, limiting the treatment of hemophiliacs.
frozen blood, but line filters are usually used.
2. CRYOPRECIPITATE, FACTOR VIII
3. ALLERGIC REACTIONS: CONCENTRATE:
Usually urticaria. Common (up to 3% of Storage: -20oC for 1 year (cryoprecipitate);
o
transfusions). Treat with antihistamina – may 4 C (Factor VIII) or room temperature for several
finish transfusion if reaction not severe, responds months
to therapy, and does not worsen when transfusion Contents: Both are made from plasma frozen
is restarted carefully. If recurrent, prescribe and within 12 hours of blood donation
antihistamine before future transfusions. Rarely Cryoprecipitate is a partly purified factor VIII
anaphylactic reactions occur and there are often in preparation containing about 50% of the factor
patients with IgA deficiency. Such patients should VIII in the original blood donation, in small
be given washed blood in the future. volume (10 mL). Each unit of cryoprecipitate (one
unit is made from one blood donation) contains
4. VASCULAR OVERLOAD: between 70 – 100 units of factor VIII ( 1 unit of
Congestive heart failure may be precipitated, factor VIII equals the amount present in 1 mL of
particularly in elderly patients with chronic plasma). Factor VIII concentrate is a more highly
anemia, given whole blood, or too rapid purified preparation of factor VIII, requiring more
transfusion of packed cells and blood components. extensive fractionation procedures; it contains a
standardized amount of Factor VIII, per vial and is
5. OTHER: more easily administered so is the preferred
Chills – rapid transfusion of refrigerated blood; product for factor VIII replacement in some
contaminated blood – very rare. situations (particularly for home care programs for
Disease transmission – hepatitis, severe hemophilia). Recently a recombinant factor
cytomegalovirus, malaria, AIDS VIII product has become available and has
Iron overload – with large numbers of transfusions replaced Standard Factor VIII concentrate.
Hyperkalemia – in renal failure
Citrate toxicity – rarely, with massive 3. ALBUMIN:
transfusions and liver failure, or prematurity. Supply: 25% albumin solution (100 mL), salt
poor, stored at 4oC
TRANSFUSION OF PLASMA AND Use: Hypoalbuminemia (usually with complicated
PLATELET COMPONENTS edema) where improvement in the patient can be
Blood and blood component therapy is influenced by reasonable expected, i.e. do not just treat a
problems of product availability and storage far more laboratory abnormality without regard to patient's
than most branches of medicine. A thorough condition, or underlying disease – albumin is
knowledge of these limitations, and continuing usually not given for cirrhosis and nephritic
communication with Blood Bank staff will ensure that syndromes. It is commonly used for supportive
Page 29 of 72
care in burn patients, and during x 10 9/L for each unit transfused, 1 hour after
hyperalimentation. transfusion in a patient of average size (eg,
Supply: 5% albumin solution (250 mL) increase platelet count by 60 x 10 9/L if 6 units
Use: This product provides a hepatitis-free given).
colloid substitute for stored plasma where volume Use: Platelet concentrate should be infused
replacement is needed (eg. Hemorrhage with promptly (1/2 an hour) through a regular blood
shock, plasma exchanged) but where coagulation transfusion set, not previously used for blood
factors are not. It is much more expensive than transfusion. Platelet response is limited by such
crystalloid solutions (eg. Normal saline, Ringer's factors as fever, splenomegaly and presence of
solution) so should only be used when colloid auto-antiobodies or allo-antibodies. In general, the
(protein) effect is really needed. presence of these antibodies makes platelet
NOTE: Albumin and immune globulin are the transfusion almost worthless, although
only blood products which are free of risk of occasionally a therapeutic response may be seen in
hepatitis transmission due to methods of a bleeding thrombocytopenic patient of this type.
production. All other products carry this risk, albeit Best results are obtained in patients who have not
small, despite extensive donor screening and viral been previously transfused, or pregnant, and where
testing; fibrinogen had a greatly increased risk thrombocytopenia is due to decreased production,
because it was pooled, as opposed to single donor rather than increased destruction or consumption
product, and is no longer issued (cryoprecipitate of platelets. Prophylactic platelet transfusion may
may be used instead to treat hypofibrinogenemia). be given to patients with severe thrombocytopenia
The factor concentrates (VIII and IX) are now (platelet count < 10 x 10 9/L) due to a disease of
specially treated ; this should prevent the viral limited duration (eg. Leukemia during
transmissions which were previously high with chemotherapy, drug toxicity, some aplastic
these products. anemias) and will be then required 2 to 3 times
weekly. Sensitization to platelet and HLA antigens
4. PLATELET CONCENTRATE: often occurs over 1 to 2 months and this limits
further transfusions of random donor platelets.
Supply: Made by differential centrifugation of Platelets obtained by platelet-pheresis from donors
blood donation less than 2 hours old. Stored for 5 who are selected for HLA compatibility may be
days at room temperature. helpful when patients become refractory to
Contents: One unit of platelet concentrate random-donor platelet transfusions.
contains about 50 to 75% of platelets present in the
original blood donation in a volume of about 50 ml 5. PENTASTARCH:
plasma. It also contains allo-antibodies (anti-A or 10% hexastarch used for the plasma volume
B). In general 6 units of platelet concentrate are expansion instead of 5% albumin. Used in
given at a time, and one would expect an increase hypovolemia, secondary to sepsis, blood loss
in platelet count in the patient of approximately 10

Page 30 of 72
PROBLEMS FOR TUTORIALS
1. ANEMIA AND CONFUSION
A 62-year-old woman was admitted to hospital after
a progressive illness of approximately 6 months
duration, characterized by progressive pallor, fatigue,
and general weakness. More recently, she had
developed shortness of breath on exertion and required
2-3 pillows at night for orthopnea. She had fallen
several times at night getting up to the bathroom, and
friends found her to be unsteady on her feet. Despite
her frequent complaints of a sore mouth she had been
eating well and had no G.I. complaints. Her past health
has been good, and she had no operations. Bone marrow: Identify the labeled structures
On physical examination, the intern found her to be
pale, slightly jaundiced and in no respiratory distress
with the head of the bed elevated. Her tongue was red 2. ANEMIA AND CONFUSION IN
and smooth. There was slight jugular venous ALCOHOLIC
distension, hepatomegaly, and pitting edema in the A 54-year-old woman was brought to the
legs. Emergency department by her daughter who found her
at the home, weak and confused. Since the patient's
Laboratory Investigations: husband had died five years ago, she was known to
Hemoglobin 50 g/L have increased her alcohol consumption and frequently
WBC 3.0 x 109/L failed to eat properly. She denied any G.I. Symptoms,
Platelets count 60 x 109/L except chronic constipation.
MCV 120 fl On physical examination she was a febrile, pale and
MCHC 350 g/L not jaundiced. Her breath smelt of alcohol and she was
Reticulocytes count 3.0% (36 x 109/L) slightly confused. There were no focal neurological
Normal (10-75 x 109/L) findings. Her tongue was smooth. On abdominal
examination there was demonstrable ascites, the liver
was not enlarged, and the spleen was palpable 4 cm
Peripheral Blood below the left costal margin. Numerous spider nevi
were present on her upper chest and shoulders, and
several bruises were noted on her legs.

Laboratory Investigations:
Hemoglobin 50 g/L
MCV 105 fL
Questions: WBC 3.6 x 109/L
1. List the clinical problems. Platelets count 86 x 109/L
2. What other physical findings related to her Reticulocytes count 40 x 109/L
unsteadiness not mentioned but which you would
expect to be present? Questions:
3. What do you expect the blood film to show? 1. List the likeliest causes for:
Describe BM features seen below? a. Macrocytosis
4. What other laboratory tests you would order? b. Pancytopenia
5. Given the diagnosis, discuss the management, c. Low Reticulocytes count
including: d. Confusion
a. Where should she be treated (hospital or
home)? Three days after admission, the patient was no
b. Approximately when you would expect longer confused and was eating well. Further
improvement to occur with regards to: investigation had been delayed by the weekend and
i. Hemoglobin on the 4th hospital day further blood tests, as well as
ii. Reticulocytes count a bone marrow aspirate, were performed. Later that
iii. Pallor day she suddenly vomited a large amount of blood
iv. Unsteadiness and became confused. Her heart rate increased to
130/minute and blood pressure fell to 70 mm Hg
systolic. A STAT Hb. was 42 G/L., prothrombin
Page 31 of 72
time was 24 seconds (INR: 2.5), and PTT was 60 A 46-year-old woman attended her doctor's office
seconds. complaining of weakness and joint pains. The
weakness had been present for several years, during
2. On the 4th hospital day, what changes would you which time she had noticed progressive joint
expect to see if the following tests were performed discomfort and stiffness mainly in her hands, elbows
that day and knees. She had been taking up to 8 aspirin tablets
a. Reticulocytes count daily for the past year and was on no other
b. Serum and RBC folate medications. Her diet was good and she had no G.I.
c. Serum B12 symptoms except indigestion related to the aspirin. She
d. Bone marrow had no past history of serious illness and was 4 years
postmenopausal.
3. You are called that evening to manage this patient On examination, she was slightly pale and not
after she vomited blood: jaundiced. There was no lymphadenopathy or
a. Explain why the hemoglobin hasn't fallen. hepatosplenomegaly. She had obvious rheumatoid
b. What would your first order for a laboratory arthritis with joint deformity of both hands.
test be?
c. What type of I.V. solution would you order Laboratory Investigations:
immediately? Hemoglobin 94 g/L
d. What further I.V. therapy would you MCV 74 fL
recommend? MCHC 310 g/L
WBC 6.3 x 109/L
3. ANEMIA AND RETICULOCYTOSIS Platelets count 230 x 109/L
A 22-year-old woman was admitted to hospital with
the chief complaints of progressive weakness, and
pallor of one-month duration. She had also noticed
some joint discomfort over recent months, but denied
any other significant complaints. Past history and
family history were non-contributory.
On physical examination, she appeared pale and
slightly jaundiced. A faint skin rash was noted over her Questions:
cheeks. There was no lymphadenopathy or 1. Classify the anemia.
hepatomegaly. Spleen was palpable 2 cm. below the 2. List 2 possible causes for the anemia.
left costal margin. No joint abnormalities were 3. List 3 most appropriate laboratory tests.
detected. 4. Assume that you conclude the patient requires iron
therapy:
Laboratory Investigations: a. What type and amount of iron treatment would
Hemoglobin 60 g/L you prescribe?
MCV 90 fL b. How long would you advise the patient to
MCHC 360 g/L continue iron treatment?
Reticulocytes count 16% (300 x 109/L) c. The patient wants to get better as quickly as
WBC 4.0 x 109/L possible and asks for either a blood transfusion
Platelets count 130 x 109/L or iron shots. Do you agree with either of these
Direct Antiglobulin (Coombs') - Positive (+++) treatments? If not, what advice would you give
to the patient to support your
Questions: recommendations?
1. List the clinical problems. d. With iron, at what rate would you expect the
2. What would you expect to see on the blood film? Hb to rise?
3. Explain why the MCV is normal.
4. List 2 reasons for the thrombocytopenia. 5. ANEMIA AND HEART DISEASE
5. In considering management of this patient: A 42-year-old man regularly visited his physician
a. What would your first choice of treatment be? for routine cardiac studies following open-heart
b. Would you recommend a blood transfusion? surgery three years ago. At that time an aortic valve
c. If the splenectomy is considered later, what prosthesis was inserted for rheumatic aortic stenosis,
precautions and advice to the patient should be and he had been anticoagulated under careful
provided? supervision since then. He had felt well until recent
weeks when he began to complain of dyspnea and
fatigue. He noticed a reddish discoloration to his urine
4. ANEMIA AND ARTHRITIS often during the day.

Page 32 of 72
On the examination, he appeared pale, but was not Mean corpuscular volume (MCV) 70u3
jaundiced. There was a tachycardia (100/minute), and a Mean corpuscular hemoglobin (MCH) 17.5 uugm
moderate (grade III/VI) early diastolic murmur Mean corpuscular hemoglobin concentration (MCHC)
maximal at the left sternal border. There was no 25%
evidence of cardiac failure. Liver and spleen were not White blood cells (WBC) 10/mm3 with normal
enlarged. differential
Sedimentation rate: 62mm/hr
Laboratory Investigations:
Hemoglobin 80 g/L CXR / ECG: normal
MCV 75 fL
MCHC 300 g/L Questions:
Reticulocytes count 2% (75 x 109/L) 1. List the clinical problems?
WBC 7.5 x 109/L 2. After reviewing CBC results, what are the next
Platelets count 300 x 109/L important simple tests you would order?
Blood film RBC fragmentation, hypochromia 3. Classify the anemia and what further tests would
you order at this stage
4. How would you manage this patient?

7. ANEMIA AND MACROCYTOSIS


42-years-old clerk presented to his G.P complaining of
inability to do his daily jogging and exercise and
Questions: reported an incident of dizziness without loss of
1. List the clinical problems. consciousness. He has been feeling gradual
2. The Reticulocytes count is not appropriate to the deterioration during the past few months. He did not
degree of anemia. Explain. have any illness and was not on any medication. There
3. Classify the anemia. was no blood loss from any site and he had normal
4. Further tests indicate iron deficiency, why do you eating habits. Apart from undergoing tonsillectomy at
think this occurred? the age of 5 years, there was no significant past history.
5. Outline your management of this patient. On examination, the patient was fully alert, pale with a
tinge of jaundice. The majority of his hair was gray
6. ANEMIA, WEAKNESS AND JOINT PAIN and upon further questioning, the patient admitted to
A 46-year old woman comes to your office having gray hair since the age of 20 years.
complaining of increasing fatigability and weakness There was no lymphadenopathy. Examination of the
for the past 4 months. She has had rheumatoid arthritis heart revealed an ejection systolic murmur over the
for 8 years and 12 to 16 tablets of aspirin per day have apex. Examination of the abdomen was normal
controlled her joint symptoms. The patient says she
had no sever pain, tenderness, redness, swelling, or Preliminary blood tests revealed the following:
heat in her joint for 2 years while on regular aspirin. WBC: 3.5 x 109 /L neutrophils 70%
However, she is less energetic and she finds she must Hb.: 83 g / L ( N: M: 140-180
stop and rest during her household work, and by the F: 120-160)
end of the day she is exhausted. She denies crying MCV 115 fl (N 80 - 95)
spells and has had no anorexia, weight loss or Platelets: 100 x 109 / L (N 150 – 400)
insomnia. She denies any history of abdominal pain, s. Iron: 30 μmol / L (males N 14 – 31)
heartburn, jaundice, and black or tarry stools. She dose s. Transferrin 2.4 g / L (2 - 4)
like to chew on ice cubes. There is no history Blood film showed the presence of macro-ovalocytes
suggestive of diabetes. Past history reveals no and hypersegmented neutrophils
additional pertinent information. Family history
reveals that she is of Italian origin and one sister was
Questions:
mildly anemic and was told she had Mediterranean
anemia. 1. What are the hematological abnormalities elicited
in the full blood count result?
On physical examination, the patient is slightly obese, 2. What may be the cause of this abnormality?
vital signs are normal. Slight pallor is noted. Cardiac, 3. What is the most likely diagnosis?
pulmonary and abdominal examination was normal. 4. How would you confirm this diagnosis?
There is no stool in the rectum for guaiac (occult
blood) testing. No signs of active rheumatoid arthritis. 8. FEVER, ANEMIA AND
RETICULOCYTOSIS
Laboratory tests: A 24 –year old Kuwaiti man is admitted to the hospital
Hemoglobin 70 gm/l, hematocrit (Hct.) 28% because of increasing fatigue following a respiratory
Page 33 of 72
infection. He felt well until 5 days ago when he dark urine implies? What gives urine this color in
developed nasal stuffiness, sore throat, and dry cough. this case?
Three days later his cough became productive of thick 4. What do you make out of the rticulocytes count?
yellow sputum, the temperature rose to 39 o C, and he Why the plasma is pink-colored?
began to note sharp pain in the left posterior thorax 5. What is you differential diagnosis? What other
with deep inspiration. His physician made clinical tests would you like to order?
diagnosis of pneumonia and prescribed Septrin and 3 – 9. CHRONIC FATIGUE AND ANEMIA
4 aspirin tablets daily for fever. Cough and fever A 28-year-old Kuwaiti lady has been complaining of
abated, but he felt progressively weaker and was chronic fatigue and known to have anemia for a long
therefore hospitalized. Past history reveals that the time on iron supplement but without any appreciable
patient had been healthy all his life. He had been told response. The first time she was told to have anemia at
that he had” yellow jaundice” as a new born but that he the age of 10 years in the range of 90 – 100 mg/L. She
had not required transfusion or other specific therapy. has been prescribed different types of iron supplements
The review of systems is entirely negative except for but with out any actual change in HB level. She is
the fact that he has noted his urine to be dark brown for single and here menstrual period is regular and scanty
the past 1 to 2 days. Family history is unobtainable. in amount. She has a brother who was told to have a
His parents are dead and his only sibling, a sister is in similar degree of anemia but she dose not know the
good health. exact nature of his problem.
On physical examination he appears acutely ill. There On examination, the patient is healthy and vital signs
is pallor, the sclera is slightly icteric. T 37.4o C, P are normal. Slight pallor is noted. Cardiac, pulmonary,
95/min, BP 120/60, RR 16/min. and abdominal examination is normal.
Respiratory movements of the left thorax are
asymmetrical with splinting on the left. There is Laboratory tests:
dullness to percussion, breath sounds are high pitched
bronchial, and many fine to medium rales are heard White blood cells (WBC) 10/mm3 with normal
over the left lung posteriorly. The heart exam reveals a differential
grade 1/6 systolic murmur heard best along the left Hemoglobin 110 gm/L, Hematocrit 37%
sternal border. Abdominal exam revealed no Mean corpuscular volume (MCV) 58 u3
organomegally. Mean corpuscular hemoglobin (MCH) 17.5 uumg
Mean corpuscular hemoglobin concentration (MCHC)
Laboratory tests 25%
HB 75 mg/l , hematocrit 25% (notice: the plasma Red cell distribution width (RDW) 15
appears pink), retics: 15%
WBC: 14.5/mm3 with 81% neutrophils, 12% bands, Peripheral blood film: microcytic, hypochromic, target
2% metamyelocytes and 5% lymphocytes. Platelets cells
count is 250.
Questions:
Blood film: reveals generally normocytic 1. How do you assess the severity of menstrual
normochromic red cells, but there is moderate period?
poikilocytosis with occasional fragmented cells and 2. Comment on the hematological picture and what is
microspherocytes. No true sickled cells are seen. There your differential diagnosis?
are many large polychromatophilic red cells and rare 3. Comment on the HB electrophoresis?
nucleated red cells. 4. If the HB electrophoresis was normal, what else
should you consider?
Urine: is clear, brown-colored, protein +, specific 5. What would you council this patient?
gravity 1.025, and glucose, ketons and urobilinogen
are negative. Dipstick test for blood is positive. No Cellulose Acetate Method (pH 8.4)
cells or casts are seen on spun sediment. Cathode - + Anode
CXR: a homogenous density involves much of the left
lower lung field. No pleural effusion seen. Renal and
liver function is normal. Serum bilirubin is 50 mmol/l
(normal< 25), 35 unconjugated, 15 conjugated.
Hemolyse C
Questions d blood S A
E
1. What does the history of jaundice as a newborn applicatio A2 D F
indicates? n
2. What are the causes of dark urine?
3. Why do we look for icterus under the upper Sample 4.5% 0.5% 95%
eyelid? What dose the Combination of icterus and
Page 34 of 72
10. A YOUNG PATIENT WITH
RECURRENT BONY ACHES
A 17-year-old male high school student presented to
the emergency department complaining of a sudden
onset right-sided chest pain. He is known to have
repeated pain attacks of similar nature that involving
different sites of his body since childhood, requiring
frequent hospitalizations. Family history revealed his
sister to have similar problem.
Physical examination revealed a pulse rate of 110/ Peripheral blood smear:
minute, Bp 110/70, and temperature of 38 °C. He was Polychromasia / Occasional nucleated RBC seen
pale and slightly icteric. Tonsils were congested. Chest Hemoglobin electrophoresis provided
examination revealed the presence of decreased air
entry on the right side. No organomegally. Questions:

Lab Investigations 1. Comment on the peripheral blood findings. What


CBC Reference Range are the Howell-Jolly bodies and what is their
WBC 15.0 (4 – 10 x 109/L) significance?
HB 87 (140 – 180 g/L)
Hematocrit 26 (42 – 50) 2. Comment on the hemoglobin electrophoresis
MCV 88 (80 – 94) findings.
MCH 30 (27 – 31 pg) 3. What is the diagnosis? And what do describe this
MCHC 340 (330 – 360 g/L) attack of pain?
RDW 17 (11.5 – 14.5 %)
Reticulocytes Count 150 (60 – 120 x 109/L) 4. What is the cause of the hyperbilirubinemia in this
s-Total Bilirubin 50 case?
s-Indirect Bilirubin 45 5. How do you manage this case?
s-Direct Bilirubin 5
6. What is the role of exchange transfusion in this
Differential WBC Count Range disease?
Neutrophils 11.5 (1.5 – 6.0 x 109/L) 7. What are the complications of this disease?
Lymphocytes 2.5 (1.0 – 3.0 x 109/L)
Monocytes 0.7 (0.2 – 0.6 x 109/L)
Eosinophils 0.2 (0.1 – 0.5 x 109/L)
Basophils 0.1 ( 0.1 – 0.3 x 109/L)

Cathode - + Anode

Hemolys C S A
ed blood E D F
applicati A2
on

70% 10% 20%

Page 35 of 72
MODULE II
WHITE BLOOD DISORDERS

OBJECTIVES
• Differentiate benign from malignant causes of leukocytosis.

• Describe the clinical presentation, plan of investigations, and be able to


interpret relevant laboratory data, so as to reach likely diagnosis and
prognosis in a patient who may present with leukocytosis and/or
lymphadenopathy.

• Explain basic molecular abnormalities underlying malignant transformation


of haematopojetic cells, and resulting in hematological malignancies.

• Plan and prescribe a rational schedule of management, based on a sound


knowledge of the mechanism of action, dose, duration and side effects of
appropriate drug therapy

Page 36 of 72
LYMPHADENOPATHY AND SPLENOMEGALY
GENERAL OBJECTIVES lymph nodes in cervical, axillary and inguinal areas
The student should have a general (occasionally epitrochlear and popliteal as well), extent
understanding of the spectrum of disorders presenting of hepatosplenomegaly, presence of tonsillar
with enlargement of lymph nodes and/or spleen, enlargement, skin lesions and bone tenderness
including methods of presentation, investigations
required to diagnose these disorders, urgency with LABORATORY ASSESSMENT
which diagnosis should be made, and treatment The qualitative assessment of white blood cells
commence and finally the prognosis and complications (leukocytes) is almost performed by automated cell
which might occur during the treatment program. counters (Coulter Counter) rather than older inaccurate
and labor-intensive counting chambers. The normal
SPECIFIC OBJECTIVES WBC count is 4.0 – 10.0 x 109/L
At the end of this section the student should be The qualitative assessment of white blood cells
able to: is most commonly performed by manual microscope
1. Describe diagnostic possibilities, given a series of techniques, with reporting of a WBC Differential
white cell counts and differentials, and/or clinical (percentage of cells assigned to each category) after
presentations of patients with enlarged lymph viewing 100 or more cells. If the WBC is too low (1.5
nodes or spleen. x 109/L) then a buffy coat preparation may be prepared
2. List the diagnostic tests required, if any, to confirm to increase the number of cells which can be viewed
the suspected diagnosis. microscopically in a reasonable period of time. Many
3. Describe the major clinical and laboratory features of the automated cell counters provide partial
of the common or important disease in this differential counts. Most commonly granulocyte and
category: lymphocyte counts. Such differentials have the
Infectious mononucleosis advantages of precision since many more cells are
Acute and chronic leukemias assessed than by manual methods, and less
Lymphomas and Hodgkin's disease technologist time is involved. However, most do not
Myeloma give full differentials and do not give a printout when
4. Differentiate between benign and malignant causes abnormal cells are present or the differential counts are
of leukocytosis and polycythemia. significantly abnormal. Fully automated differential
5. Describe the expected outlook for patients once counters require a large volume of tests to be cost
any of the above diseases are diagnosed effective and are not as helpful in settings where a
6. List the possible treatment options, when large percentage of the differential are abnormal.
available, for theses diseases, and describe the The bone marrow may provide information
risks / benefits expected with these treatments. about granulocyte production, and presence of
7. describe the management of the common infiltrative disorders such as leukemia, lymphoma and
complications which might arise during the course myeloma. The normal sequence of granulocyte
of these disorders; these complications include development is as follows:
bone marrow suppression, local effects of enlarged
lymph nodes and spleen, pain and hyperviscosity. Myeloblast → promyelocyte → myelocyte →
8. Describe the role of blood component therapy in metamyelocyte → band → neutrophil
the management of these patients, including type
of component, rate of administration, and expected Normally there are 3 – 4 times as many
responses granulocyte precursors, than erythroid precursors, in
the bone marrow (M/E ratio 3/1). This reflects the
CLINICAL ASSESSMENT presence of a large granulocyte pool or reserve, and
Particular points in history include a search for also the short life span of granulocytes compared with
sites of local infection, or if the lymphadenopathy is erythrocytes.
generalized and/ or splenomegaly is present, particular
attention must be paid to systemic symptoms (fever, The maturation of granulocytes in the marrow
night sweats, pruritus, weight loss, anemia and is sequential. If there are increased numbers of less
jaundice), medications, travel history, and presence of mature forms, then this is termed "shift-to-the-left". It
other diseases (eg. Rheumatoid arthiritis, lupus, etc). could represent a reactive process (granulocytic
hyperplasia) or a malignant process (leukemia and the
Physical examination should include a careful morphological distinction may be difficult.
documentation of the size and consistency of enlarged
Page 37 of 72
BENIGN LEUKOCYTE DISORDERS
GRANULOCYTOPENIA occur due to granulocyte consumption or complement
This term refers to neutropenia, or reduction in activation, and depletion of the marrow granulocyte
the absolute neutrophil count; decreased in the pool. It is often an ominous sign. In hypersplenism,
eosinophils and basophils cannot be accurately there is usually a pancytopenia due to splenic pooling
measured, and are of no practical importance. The of hematopoietic cells and hypercellular marrow, with
severity of the neutropenia is usually graded as correction of blood counts after splenectomy.
follows: Severe neutropenia may occur with
Normal neutrophil count 2.5 – 7.5 x 109/L Rheumatoid arthritis (Fety's syndrome) usually with
Mild neutropenia 1.0 – 2.5 x 109/L splenomegaly as well. This may be due to auto-
Moderate neutropenia 0.5 – 1.0 x 109/L antibodies against granulocytes, which also likely
Severe neutropenia < 0.5 x 10 9/L occur in some patients with lupus, but laboratory
A serious risk of spontaneous infection is techniques are not presently available to detect these
usually not present until severe neutropenia develops. antibodies.
The management of neutropenia involves
CLASSIFICATION OF NEUTROPENIA discontinuing any drugs which may be associated,
1. Decreased granulopoiesis – marrow infiltration / correcting any vitamin deficiency or nutritional
fibrosis, drugs, radiation problems, and if possible, controlling any underlying
2. Ineffective granulopoiesis – megaloblastic anemia disease. If hypersplenism cannot be reversed by other
3. Decreased granulocyte survival – hypersplenism, means, and if the cytopenias are severe enough to
severe infection, immune warrant the risks involved, then a splenectomy may be
DECREASED GRANULOPOIESIS indicated.
This may be associated with decrease in other
marrow elements causing pancytopenia (see aplastic LYMPHOPENIA
anemia, leukemia, myelofibrosis, etc). When May also occur (absolute lymphocyte count <
granulocytopenia occurs selectively due to decreased 1.5 x 109/L) due to drugs (steroids), hereditary
granulopoiesis, it is most frequently caused by drugs. immuno-deficiency disorders or associated with some
Even then, many of these may be due to drug-related other diseases, such as Hodgkin's disease & HIV
granulocyte antibodies which are not detectable by infection. It may be present, usually to a lesser degree
current techniques. than neutropenia, in association with the pancytopenia
Some drugs, such as chemotherapeutic agents, of aplastic anemia, hypersplenism, etc.
will predict cause neutropenia, but usually also
thrombocytopenia and anemia. These are usually slow GRANULOCYTOSIS
in onset and dose-dependent. Other drugs may When the numbers of granulocytes are
unexpectedly cause neutropenia, often severe and increased in comparison to the numbers of
sometimes fatal, in a previously sensitized patient, or lymphocytes, but in absolute terms, then this is called
by unknown mechanisms (idiosyncratic reaction). "relative granulocytosis". It is usually seen with the
Many drugs have been implicated, similar to those lymphopenias described previously. When the actual
which cause aplastic anemia. The major groups include number of granulocytes is increased (and this is
anti-inflammatory agents, anticonvulsants, antibiotics, calculated from the percentage of granulocytes and the
antimal arials, antithyroid drugs and phenothiazines. total WBC count) this is termed "absolute
The neutropenia may last for days / weeks. granulocytosis" and these conditions are now
considered further, depending on which granulocyte is
INEFFECTIVE GRANULOPOIESIS increased.
This is seen most often in megaloblastic a. Neutrophilia (neutrophil granulocytosis)
anemias, characterized by a pancytopenia but a An absolute neutrophil count greater than 7.5 x
hypercellular megaloblastic bone marrow; the 109/L is called neutrophilia; the causes which
precursors of all cells lines dies in the marrow cavity would be considered are:
before being released. Thus, ineffective hematopoiesis i. Controlled proliferation
is corrected by appropriate therapy such as B12 or This is a normal response to infections
folate vitamin treatment or in the case of alcohol (mainly bacterial), trauma, inflammatory
excess cessation of alcohol consumption. diseases an d even neoplasm. Leukocytosis up
to 50 x 109/L is common and elevations as high
DECREASED GRANULOCYTE SURVIVAL as 100 x 109/L can occur. Above this level,
In severe bacterial infections such as lobar however, a leukemic process will almost always
pneumonia, and various septicemias, neutropenia can be present. There will often be left-shift with

Page 38 of 72
reactive neutrophilia but this will seldom extend As noted previously, children have a large
as far back as the myeloblast and promyelocyte proportion of lymphocytes on WBC differential than
stages. When the reactive leukocytosis is so adults, and frequently these lymphocytes appear
extreme and/or left-shifted as to resemble a atypical, or reactive, in response to the
leukemia, it is termed "leukemoid" reation. immunological challenges and frequent viral
ii. Uncontrolled proliferation infections during childhood.
This represents a malignant myelproliferative Absolute lymphocytosis in adults requires a
process such as leukemia, myeloid metaplasia lymphocyte count greater than 3.5 x 10 9/L. This will
or polycythemia, rather than reactive or usually be due to some underlying disease, usually a
secondary, neutrophilia. viral infection, but if it is persistent, without
iii. Storage pool activation underlying cause, then it may represent an early stage
Roughly 1/3 of the total granulocyte mass is of chronic lymphocytic leukemia – the distinction
present as a storage pool in the marrow. This cannot be made on morphological grounds alone. Up
reserve may be rapidly mobilized in response to to 5% of lymphocytes in adults and 10% in children
many acute traumatic and inflammatory may appear slightly atypical or reactive; these
processes resulting in a prompt neutrophilia in lymphocytes are larger with less mature nucleus, and
peripheral blood. The mature granulocytes are the cytoplasm is often basophilic, and in large amount
depleted in the marrow for several days until so that the cell membrane appears compressed by
granulocyte hyperplasia occurs. surrounding erythrocytes on the blood film.
iv. Marginal pool activation Causes for lymphocytosis of moderate to
2/3 of the total granulocyte mass is present marked degree, include infectious mononucleosis,
outside of the marrow, and this is divided equally cytomegalovirus, infectious lymphocytosis, Bordetella
between a central pool (1/3 of total mass) and a pertussis (whooping cough), toxoplasmosis, HIV and
marginal pool (the last 1/3). The number of hypersensitivity reactions. The infectious
marginating neutrophils may be changed acutely lymphocytosis of whooping cough is distinctive
due to catecholamines (any stressful situation) or because the lymphocytes are all small lymphocytes,
by migration inbibition with steroids, causing the not atypical, and the lymphocyte count may reach
neutrophils to demarginate and be counted in the extreme degrees (50 -100 x 109/L) resembling chronic
central pool. The result is an absolute lymphocytic leukemia, which is not seen in children.
neutrophilia on WBC counting, even though the For practical purposes, major degrees of atypical
number of circulating granulocytes hasnot lymphocytosis (20 – 40% of WBC's) in adults will be
actually changed. due to infectious mononucleosis (EB virus) or
b. Eosinophilia cytomegalovirus; the latter may cause an identical
Absolute eosionophilia (0.4 x 109/L) is clinical picture but with a negative mono test, and is
most commonly associated with the following much less commonly diagnosed. Infectious
disorders: mononucleosis characteristically presents with fever,
• Allergies – allergic rhinitis, asthma, etc sore throat and cervical lymphadenopathy; jaundice is
• Parasitic infections – amebiasis, nematodes, uncommon (5%) but laboratory evidence of hepatic
trematodes, etc dysfunction is common (elevated transaminase in over
• Dermatitis – dermatis herpetiformis, eczema, 80%). Skin rash may occur, particularly if these
etc. patients are treated with ampicillin. Occasionally
hemolytic anemia (cold agglutinins) and
• Hypereosinophilic syndromes – Loefler's thrombocytopenia may be prominent features.
syndrome, polyarteritis, etc Splenomegaly is present in half of the cases but must
• Neoplasms – various carcinomas, Hodgkin's be carefully examined because of the risk of splenic
disease,etc. It is extremely rare for rupture. The diagnosis is confirmed by a positive
eosinophilia to occur as a primary monospot test in 95% of cases, and the treatment is
myeloproliferative disorder (eg. Leukemia) usually only supportive (bed rest, analgesics, or
c. Basophilia antipyretics) Steroids are indicated only for serious
Increase in basophils is rare, and virtually complications (CNS involvement, pharyngeal
always part of other leukocyte abnormalities in obstruction, sever thrombocytopenia) Sometimes the
myeloproliferative diseases (CGL, myeloid lymphocytes are fragile, and are ruptured during the
metaplasia, polycythemia) preparation of the blood film; they are then referred to
d. Monocytosis (0.8 x 109/L) as "smudge cells". These are most common in chronic
Elevation of monocytes is uncommon. If may lymphocytic leukemia but can also be seen with
occur with some chronic infections, preleukemia reactive lymphocytosis (eg. Viral infection) so are not
or monocytic leukemia. diagnostic of C.L.L
e. Lymphocytosis

Page 39 of 72
HEMATOPATHOLOGY TESTING
DIAGNOSIS OF ACUTE LEUKEMIA that is involved in the rearrangement of
Classification of acute leukemias according to FAB immunoglobulin and T cell receptor genes.
criteria requires morphologic examination of a bone If a leukemia is atypical (i.e. of T cell origin)
marrow aspirate smear. This is required to obtain a additional antibodies can be used to further
blast count, as well as to determine the blast characterized the leukemic cells. If the leukemia cells.
morphology, the number and types of maturing cells, If the leukemia has a mature B cell phenotype (i.e.
and whether dysplastic features are present. expresses CD19 and/or CD10 but not TdT, as in
Burkitt’s leukemia), antibodies recognizing κ and λ
Cytochemical Stains immunoglobulin light chains are added to the panel to
It is often impossible to determine whether the establish clonality. If the leukemia is of plasma cell
blasts in an acute leukemia are of myeloid or lymphoid origin (i.e. multiple myeloma), staining for
origin from morphology alone. For this reason, cytoplasmic immunoglobulin is performed by
cytochemical staining is performed. immunofluorescent microscopy or flow cytometry.
This is done since plasma cells generally lose cell
Stain Cells stained surface expression of the useful markers, including κ
and λ.
Myeloperoxidase Myeloid

Non-specific esterase Monocytes, blasts


in monocytic leukemia

Periodic acid-Schiff Normal granulocytes,


blasts in lymphocytic
Leukemia ("string of
pearls"), abnormal
Erythroblasts in DIAGNOSIS OF LYMPHOMA AND
erythroleukemia LYMPHOPROLIFERATIVE DISORDERS
("chunky")
Most lymphomas are characterized by
Tartrate-resistant Hairy cell leukemia cells immunophenotyping cells from the lymph node or
acid phosphatase (TRAP) other tissues in suspension or frozen tissue sections.

Leukocyte alkaline High in leukemoid Flow Cytometry


phosphatase reactions , low in CML Flow cytometry is the method of choice for
characterizing lymphomas involving the peripheral
Flow Cytometry blood or bone marrow. Since most lymphomas are of
Flow cytometry is used to determine the cell type of mature B cell origin, finding a B cell population that
leukemic blasts. expresses exclusively κ or λ light chains on the cell
surface is sufficient for the diagnosis of malignancy.
Stem Cell CD34 Actual classification requires morphologic
examination, as well as determining the expression of
Myeloid CD13, CD33 other cell surface markers. The following table lists the
antibodies used for the characterization of lymphomas:
B cell CD10 (CALLA), ___________________________________________
CD19 _
Cell Surface Marker C ells Identified
T cell CD2, CD5, CD7
CD5 Pan-T cell marker; also expressed
Megakaryocyte CD61 on some B cell lymphomas (small
lymphocytic lymphoma/CLL, mantle
In addition to the above markers, the "leukemic cell lymphoma)
panel" includes immunofluorescent staining for
terminal deoxynucleotidyl Tranferase (TdT). This is an CD19, CD20 Pan-B cell markers (CD20 is low or
enzyme present in the nuclei of immature lymphocytes absent in SLL/CLL)

Page 40 of 72
κ, λ Immunoglobulin light chains, used to chromosome 22. The resulting fusion protein, BCR-
determine clonality in B cell ABL is thought to cause neoplastic transformation.
lymphomas This chromosomal translocation can be detected by
DNA blot analysis using a probe for the BCR gene.
CD2 Pan-T cell marker, also expressed on This particular probe easily detects the translocation if
NK cells as few as 1% of the cells present contain the
Philadelphia chromosome. This probe detects the
CD4, CD8 T cell subsets translocation in greater than 90% of CML. Because the
breakpoint is different, the probe only detects about
CD1 mature T cells, Langerhans cells 33% of Philadelphia chromosome-positive ALL. This
test can be used to make the differential diagnosis of
CD3, CD7 Pan-T cell markers CML vs. other reason for leukocytosis or
thrombocytosis. It can also be used to detect minimal
CD16/56, CD57 NK cell markers residual disease following bone marrow
transplantation, although it is not as sensitive as PCR.
CD25 IL2 receptor; present on In addition to increased sensitivity, this method will
activated T cells, some T cell allow detection of both types of t(9;22) breakpoints
malignancies, and hairy (i.e. CML vs. ALL types). Detection of t(14;18)
cell leukemia In up to 90% of follicular lymphomas, a
translocation between chromosomes 14 and 18 can be
found. This translocation results in the insertion of the
MOLECULAR STUDIES Bcl-2 gene from chromosome 18 into the
immunoglobulin locus on chromosome 14, Bcl-2 is a
Immunoglobulin and T Cell Receptor Gene mitochonderial protein that regulates apoptosis or
Rearrangement programmed cell death; the function of Bcl-2 appears
As a part of normal development, lymphocytes to be the prevention of cell death by apoptosis. The
rearrange their immunoglobulin (B cells) or T cll t(14; 18) results in over expression of Bcl-2 allows
receptor genes (T cells). This results in alarge cells to survive longer, and thereby accumulate
repertoire of receptors for antigen recognition. It also additional mutations which lead to neoplastic cell
allows determination of B or T cell clonality, since transformation.
each lymphocyte has a different DNA structure. In a
mixed population of lymphocytes, no one lymphocyte The t (14; 18) can be detected by PCR. The main
clone is of sufficient number to be detected by DNA use of this test is to determine whether peripheral
blot analysis, however when one clone predominates, blood, bone marrow, or various stem cell
its receptor gene rearrangement can be detected. When transplantation products are contaminated by
a single clone is detectable, this is generally equivalent lymphoma cells. The test can also be used to make a
to lymphoid malignancy, although there are a number diagnosis of follicular lymphoma, particularly on
of specific conditions where clonality does not needle biopsy specimens where follicular architecture
necessarily mean malignancy. may not be apparent.

The technique used for detection of gene


rearrangements is DNA blot analysis. Genomic DNA
is isolated from a tissue biopsy, peripheral blood, or
bone marrow specimen.
Recently the polymerase chain reaction (PCR) has
been used to detect immunoglobulin heavy chain gene
rearrangement.

BCR Gene Rearrangement


Chronic myelogenous leukemia (CML) is caused
by a translocation between chromosomes 9 and 22 (the
Philadelphia chromosome). The result of the
translocation is the juxtaposition of the c-abl proto- SKY allows for direct visualization of a specific chromosomal
oncogene on chromosome 9 with the BCR gene on abnormality (numeric and translocation

Page 41 of 72
Result of FISH

Normal cell MCL – lymphoma cell

Microarray technology allows for a large number of genetic


abnormalities to be screened on a single chip that is then scanned
and analyzed by a computer
• FISH allows for direct visualization of a specific chromosomal
abnormality.
• FISH studies are less sensitive than PCR-based methods, but can
detect abnormalities,
• such as monosomies and trisomies, that cannot be studied by PCR
analysis.

HODGKIN’S AND NON-HODGKIN’S


LYMPHOMAS
HODGKIN'S DISEASE Diagnostic Reed-Sternburg (RS) cell

Thomas Hodgkin’s
Immune stain:

Hodgkin's disease is not common (annual CD15 & CD30 positive in classical HD
incidence about 2 per 100,000) but it has generalized LPHD:
intense interest because of its predilection to affect
young people, and its potential for cure with either
radiation or chemotherapy. It is generally considered to
be a type of lymphoma but clearly differentiated from
the other "non-Hodgkin's" lymphoma to be discussed abundant lymphocytes
later.
Common clinical features include fever, night
sweats, pruritus, weight loss, lymphadenopathy and
splenomegaly. Tissue diagnosis is mandatory, and
achieved by lymph node biopsy or biopsy of another
involved organ (eg. Liver, lung)
Nodular Sclerosis: nodularity / RS cells abundant
Pathological classification:
LDHD: few lymphocytes seen
The W.H.O. classification will be used for
classification of Hodgkin's disease
- Hodgkin's Disease, Lymphocyte
predominance (LPHD)
- Classical Hodgkin's disease
Nodular Sclerosis (NSHD)
Mixed cellularity (MCHD)
Lymphocyte Depletion (LDHD)
Lymphocyte-Rich (LRHD)

Page 42 of 72
MCHD: pleomorphism ,
PET Scan

mixed lymph, eosinophils, plasma cell.

The clinical stage or extent of the disease however is


more important in planning therapy and influencing
FDG PET scan in a newly diagnosed lymphoma.
prognosis. The following simplified stages are Widespread nodal disease is shown above and below
currently used: the diaphragm. Spleen (arrowheads) is diffusely
Stage I – disease in one lymph node region hypermetabolic, suggesting lymphomatous
Stage II – disease in two or more lymph nodes but involvement.
confined to one side of diaphragm
Stage III – disease confined to lymph nodes or 5. Bone marrow aspirate and biopsy are done as part
spleen and present on both sides of of staging work up to exclude bone marrow
diaphragm involvement. This is more likely if there are 'B'
Stage IV – disease outside confines of lymph symptoms and hematological abnormalities.
nodes or spleen. (i.e. involves organs) The principles of management of HD include the
following:
The presence or absence of systemic symptoms are a. LPHD : local radiotherapy
also important; these include: b. Classical HD stages I, IIA include 4
A– absence of fever, night sweats or courses of chemotherapy and involved
weight loss (10% of body weight) field radiotherapy
B– presence of any of the above. c. Advanced stages (IIB, III, IV) includes
systemic chemotherapy of 6 – 8 cycles.
The procedures required to stage patients with Local radiotherapy may be given for initial
Hodgkin's disease vary somewhat from centre to bulk (tumors > 10 cm)
centre, but generally include the following: The potential for cure exists for all stages and
1. Routine hematology and biochemical tests all therapies but is much higher for earlier stages and
including liver function asymptomatic disease. It is important to optimize
2. Chest X-ray – PA and lateral; CT chest or treatment to minimize side effects. Chemotherapy,
tomogram if indicated induces considerable morbidity; nausea and vomiting,
anorexia, hair loss and long term can lead to sterility.
However, the most commonly used protocol is known
3. CT Scan of abdomen as ABVD (Adriamycin, Bleomycin, Vinblastin ,
DTIC) has less toxicity and is the most commonly
used.. It has low infertility rate and less leukomogenic
effect. Currently cure rates are high > 70 % for patients
with good prognosis. However, the relapse rate is over
40% in patients with bad prognosis (advanced stages).
For those who relapses a high dose chemotherapy and
4. Nuclear scan: autologous peripheral stem cell transplant is offered.
Gallium scan: shows active uptake in the lymphoid The progression free survival is 50 % at 5 years.
tumor. More recently PET scan is more utilized as
it is more sensitive and more specific LYMPHOMAS
Gallium uptake –Anterior The “non-Hodgkin's" lymphomas are a somewhat
mediastinal mass variable group of malignant neoplasms primarily
involving lymph nodes and/or spleen, although almost
any organ can be involved. The clinical presentation is
virtually identical to those of patients with Hodgkin's
disease. Lymphomas are more likely to affect extra
nodal sites e.g. gastrointestinal tract, than Hodgkin's
disease in which this presentation is uncommon.

Classification of Non-Hodgkin’s Lymphoma


– B Cell Neoplasms
• Precursor B-cell neoplasms
Page 43 of 72
• Mature B cell neoplasms The NHLs can be divided into two prognostic groups:
– T and NK Cell Neoplasms the indolent lymphomas and the aggressive
• Precursor T cell neoplasms lymphomas.
• Mature T and NK cell Indolent NHL types have a relatively good prognosis,
neoplasms with median survival as long as 10 years, but they
Examples of these cases: usually are not curable in advanced clinical stages.
• Diffuse large B cell lymphoma (DLBCL) Early stage (I & II) indolent NHL can be effectively
• Follicular lymphoma treated with radiation therapy along. Most of the
• Mucosa associated lymphoid tissue (MALT) indolent types are nodular (or follicular) in
• Burkitt lymphoma morphology.
• CLL/SLL The aggressive type of NHL has a shorter natural
• Cutaneous T-cell lymphomas MF/Sezary history, but a significant number of these patients can
be cured with intensive combination chemotherapy
regimens)
In general the well differentiated histological types and
follicular (nodular) pattern have an indolent course. At
present these cannot be cured but have a relapsing
course and survive 7-10 years. Some live considerably
longer.
The high grade lymphomas have a more
CLL CLL aggressive course. Previously such lymphomas had a
bad reputation with early death. Combination
chemotherapy with regime that usually contain
cyclophosphamide, adriamycin (Doxorubicin), as well
as other drugs have changed this outlook. The moast
Follicular NHL common chemotherapy protocol used is CHOP
(Cyclophosphamide, Hydrodunarubicin, Onconvin,
Presnisone). Now approximately 50% of patients with
diffuse large cell lymphomas may have long term
remissions and probable cure. Recently it has been
shown the use of monoclonal antibody therapy
increases the chances of response and cure rate. The
most widely effective and popular monoclonal
antibody is an anti CD20 monoclonal antibody, known
as Mabthera or Rituxan. Currently, the use of anti
Diffuse large B cell Lymphoma express CD 20 antigen CD20 monoclonal antibody in addition to
chemotherapy is the standard of care for the most
CD20 + Large B- cells common aggressive lymphoma (diffuse large B cell
lymphoma).Once relapse occurs, patients usually
undergo a high dose chemotherapy and autologous
stem cell transplant.. Results are better too in the
treatment of two very aggressive high grade
lymphomas with increasing evidence for possible cures
Burkitt lymphoma with starry sky appearance in this group, namely Lymphoblastic lymphoma and
Burkitt’s lymphoma. The current therapy for these two
lymphomas include intensive systemic chemotherapy
and CNS directed prophylaxis.
Most of the indolent lymphomas, particularly
the follicular and small lymphocytic lymphoma, have
reached a widespread stage at the time of presentation.
Bone marrow and liver are frequently involved in
addition to generalized lymphadenopathy and
splenomegaly.
Mucosa associated lymphoid tissue Follicular (nodular) lymphomas of insidious
MALT) lymphoma of the stomach onset without significant symptomatology do not need
treatment and many years may pass until treatment
becomes indicated. When treatment is required, gentle
chemotherapy with Chlorambucil will usually suffice
for elderly patient. However, for younger patients a
Page 44 of 72
systemic chemotherapy and monoclonal antibody Radiptherapy may also be used in combination with a
therapy may produce better results and long chemotherapy programme to treat bulky disease. It
remissions.. For localized treatment radiotherapy may often plays an important role in treatment of relapsed
be indicated but this is not common as most cases have disease when chemotherapy has failed or to treat a
generalized disease or are of more aggressive type palliative symptomatic disease when cure is not
such that chemotherapy is the treatment of choice. possible.

Common chromosome translocations in non-


Hodgkin lymphoma
Chromosome
abberation Lymphoma Genes involved
t(14;18)(q32;q21) Follicular lymphoma BCL-2, IgH
Diffuse large B-cell lymphoma
t(8;14)(q24;q32) Burkitt lymphoma C-MYC, IgH
t(8;22)(q24;q11) Burkitt lymphoma C-MYC, IgL
t(2;8)(p11;q24) Burkitt lymphoma C-MYC, IgK
t(11;14)(q13;q32) Mantle cell lymphoma CCND1, IgH
t(11;18)(q21;q21) Marginal zone/extranodal MALT API2, MALT1
t(14;18)(q32;q21) Marginal zone/extranodal MALT MALT1, IgH
t(1;14)(p22;q21) Marginal zone/extranodal MALT BCL-10, IgH
t(1;2)(p22;p12) Marginal zone/extranodal MALT BCL-10,IgK
t(3;14)(p13;q32) ”de novo” Diffuse large B cell BCL-6, IgH
t(3;22)q27;q11) ”de novo” Diffuse large B cell BCL-6, IgL
t(2;3)(p12;q27) ”de novo” Diffuse large B cell BCL-6, IgK
t(2;5)(q23;q35) Anaplastic large cell T/null ALK, NPM

Page 45 of 72
MYELOMA
Myeloma is a disease caused by malignant b. Bone marrow
proliferation of plasma cells. It usually presents as a - 10 – 20% plasma cells usually required for
bone disease, with bone pain, but multiple other diagnosis of myeloma. Multinucleated and
presentations are possible due to many complications immature forms are common.
which may occur. c. Biochemical tests for
Most cases of myeloma are due to clones of - total protein (increased) and albumin
plasma cells producing immunoglobin G (IgG), but (decreased)
IgA myeloma is also common – IgD and IgE myeloma - hypercalcemia
are rare. Myeloma must be differentiated from: - hyperuricemia
- renal function
1. Benign Monoclonal Gammopathy – long-standing - β-2 microglobulin
non-progressive monoclonal increase in - LDH
immunoglobulin without marrow plasmacytosis or - C- reactive protein
bone lesions. Usually the Ig elevation is mild (25 - Cytogenetic studies
g/L) and urinary light chain proteinuria is present. - serum protein electrophoresis – detects and
2. Waldenstrom’s macroglobulinemia – this quantitates the monoclonal protein; about 15%
uncommon disorder presents with monoclonal however, have hypogammaglobulinemia.
elevation of IgM; in contrast to myeloma, bone
disease is usually absent, whereas
lymphadenopathy and splenomegaly may occur.
The cellular infiltrate is heterogeneous
(lymphocytes, plasma cells and plasmacytoid
lymphocytes) Hyperviscosity symptoms may be
prominent. The disease is usually slowly
progressive; if therapy is needed then
Chlorambucil will usually suffice.

Myeloma – the following presentations are worth


noting. - Urine protein (24 hour) and protein
CLINICAL electrophoresis – quantities the amount of light
As noted above, bone pain due to osteolytic bone chain proteinuria (routine urinary dipstick test
lesions with or without pathological fractures or for protein does not detect light chains).
vertebral compression fractures from osteoporosis or d. Immunology tests
osteolytic lesions may all occur. Symptoms of anemia - Ig quantitation Immunisation electropheris –
(fatigue, pallor), leucopenia (infections) or confirms Ig type (IgG, IgA, etc) and light
thrombocytopenia (bruising or bleeding) may all be chain (kappa or lambda)
present due to myeloma replacement of bone marrow. - Urinary light chains – selective kappa or
Renal failure may result from myeloma kidney (light lambda light chain proteinuria
chain, or Bence-Jones protein, precipitation), e. X-rays
amyloidosis, hypercalcemia, hyperuricemia or - skeletal survey to detect osteolytic lesions or
infection. Amyloidosis may also cause tongue diffuse osteoporosis. X-rays of skull, spine and
enlargement, arthropathy, hepatosplenomegaly, pelvis are particularly important, but ribs and
purpura, carpal tunnel syndrome, etc. Hypercalcemia long bones can be involved and should be x-
may induce polyuria, polydipsia, vomiting and rayed if clinically indicated. Note that alkaline
confusion. phosphatase is usually not elevated, and bone
LABORATORY scans are usually normal because the bone
lesions are almost totally osteolytic in nature.
Bone marrow MANAGEMENT
Infiltrated with
plasma cells
1. CHEMOTHERAPY
Melphalan and Prednisone given for 4 days
each month are helpful in palliative treatment, causing
a. Blood picture control or regression of myeloma in most, but not all,
- anemia, leucopenia and thrombocytopenia patients. Morbidity from chemotherapy is minimal. It
may be continued as long as patient lives, or stopped
Page 46 of 72
after improvement is complete, or stable, to be 4. Treatment of infection: Multiple myeloma and
restarted at the time of relapse. Cyclophosphamide its treatment including chemotherapy and
may be substituted for melphalan if it is no longer transplantation render patients extremely
effective. There is definite increase of leukemic vulnerable to infection. Prompt treatment of
transformation in patient receiving alkylating agent infection and in selected cases prophylactic anti-
chemotherapy but the improved quality of life and infective therapy reduces morbidity and
survival warrant this risk. morbidity.
Treatment of multiple myeloma in
symptomatic patient is dictated by age and presence of 5. SPECIAL SITUATIONS
co-morbidities. For patients aged below 70 years initial • Amyloidoisis is usually, but not always,
induction by combination chemotherapy of Vincristine, unresponsive to any treatment.
Doxarubicin, Dexemethasone (VAD) 3-4 cycles • Hypercalcemia – must be promptly and
followed by high dose chemotherapy and autologous aggressively treated with large volumes of
stem cell transplantation is the treatment of choice and intravenous saline, steroids, and chemotherapy
results in longer progression free survival and overall or biphosphonates.
survival compared to conventional chemotherapy.
• Allogeneic bone marrow transplantation can
cure multiple myeloma, however, high rates of
treatment related mortality and poor overall
2. RADIOTHERAPY
survival precludes its routine use. Low
Excellent treatment for symptomatic isolated
intensity allogeneic bone marrow
bone involvement, unresponsive to chemotherapy.
transplantation following cytoreductive
Radiotherapy is used in patients with cord compression
chemotherapy, autologous transplantation may
and in prevention of impending pathological fractures.
however be used in high risk cases. Patients
not eligible for high dose therapy should be
3. SURGERY
treated with combination of Melphalan &
For pathological fractures
Prednisolone or Dexamethasone with
Thalidomide. Thalidomide and its analogoue
4. SUPPORTIVE CARE
lenalidomide has antiangiogenic, antimyeloma
1. Biophosphonates: Bone involvement in multiple
and immunnomodulatory effect and has been
myeloma is a major reason for morbidity of the
found to be effective in newly diagnosed
disease. Biophosphanates inhibit bone
myeloma as well as in relapsed patients.
resorption. Pamidronate or the newer more
Bortezomib (valcade), a proteosome inhibitor
potent Zoledronate are usually used to prevent
has been found to be effective in relapsed
skeletal events. Ideally such treatments should
multiple myeloma and is under trial in patients
be combined with calcium and hormone
with newly diagnosed multiple myeloma.
replacement in patients with no contradiction to
their use. • Renal failure – treatment of hypercalcemia,
2. Erythropoetin: Anemia is frequent in patients hyperuricemia and the myeloma are all
with multiple myeloma. Recombiant human important to reverse as much of the renal
erythropoietin decreases transfusion failure as possible. If renal failure is
requirement, increases means hemoglobin, established and severe, then careful medical
improves quality of life and performance status management of the azotemia, and electrolytes,
in aneamic myeloma patients. Erythropoetin will be needed; dialysis is sometimes
may be used in weekly or three times weekly employed if the life expectancy and general
schedules condition of the patient warrants this.
3. Vertebroplasty and Kyphoplasty: Painful The treatment of myeloma improves the
vertebral compression is a major cause of quality of life (decreases bone pain, improves
morbidity in multiple myeloma patients. blood counts, etc) and prolongs life (median
Vertebroplasty and Kyphoplasty helps in survival is now 3 – 4 years compared to 1 year
relieving pain in most patients untreated). Current treatment does not cure the
disease.

Page 47 of 72
LEUKEMIAS
ACUTE LEUKEMIA nodes (including mediastinal) and splenomegaly,
The acute leukemias are not common (annual whereas these features are usually absent in the
incidence rate of 3.5 / 100,000) but a major cause of commoner AML in adults. Acute monocytic leukemia
morbidity and mortality particularly in young people presents typically with gingival enlargement from
(acute lymphoblastic leukemia is the commonest monoblast infiltration.
cancer in children). Although the incidence is not
increasing, the prevalence is, since many cases who MANAGEMENT
previously would have died quickly are now achieving
prolonged remissions with chemotherapy and CHEMOTHERAPY
supportive care. The initial goal of chemotherapy is remission
The hallmark of the acute leukemias is the induction ( a complete remission has occurred when
characteristic proliferation of immature (blast) cells in there is no evidence of leukemia in the blood and < 5%
the bone marrow, and often in the peripheral blood blast cells in the bone marrow) This is readily achieved
where WBC counts can be increased up to 100 x 109/L in children with ALL (95%) but only in about 60%
or more. If the diagnosis is obvious from the WBC adults with AML. Combination chemotherapy is
differential (large number of blast cells) then a bone required with various protocols (drugs such as
marrow is not essential, but if there is any doubt when vincristine, prednisone, L-asparaginase, methotrexate,
a bone marrow will be required to demonstrate the 6-Mercaptopurine, adriamycin, cytosine, arabinoside,
involvement, or often complete replacement by rapidly and 6-thioguanine) are used in cyclic fashion to bone
proliferating blast cells. The blast cells fail to marrow patient tolerance. In the treatment of AML this
differentiate into more mature forms (i.e. granulocytes, aggressive chemotherapy is continued for two or more
lymphocytes or monocytes) and inhibit maturation of courses after complete remission is achieved for
normal clones of marrow cells. The acute leukemias consolidation. Maintenance treatment is not usually
are divided into the following categories. used as it has not been proven to prolong survival in
AML. In treatment of ALL following induction of
5. Acute myeloblastic leukemia (AML) – variants remission, consolidation therapy is given which is
include acute myelomonocytic leukemia followed by maintenance treatment. The total time
(AMML), promyelocytic leukemia and treatment undertaken is 2 – 3 years. Most maintenance
erythroleukemia treatments are given on an outpatient basis and patients
6. Acute lymphoblastic leukemia (ALL) – variants return to regular routines of daily living.
include acute prolymphocytic leukemia, In childhood ALL, the risk of CNS
undifferentiated leukemia involvement or meningeal leukemia is so high ( and
7. Acute monocytic leukemia not counteracted by systemic chemotherapy) that these
patients receive prophylactic intrathecal chemotherapy
Morphological criteria (plus cytochemistry) used and is a subset cranial radiation to prevent CNS
to be the prime method of differentiating the various relapse.
types of leukemia. Now with the use of monoclonal The outlook for childhood ALL is much
antibodies, cell surface antigens can be identified, improved with modern chemotherapy protocols with
enable a more accurate differentiation of acute the prospect of cure in up to 70% of cases. In adults
leukemias. This is particularly important in results are not so encouraging. A median survival of 18
determining the type of acute lymphoblastic months for AML responders and 30% long term
leukemias. Cell surface marker analysis of ALL now survivors (5 years) can be achieved. For ALL recently
determines the type of treatment used and probable 25-40% long term survival has been achieved.
prognosis. Morphological criteria are still important Morbidity during aggressive induction
and both techniques are necessary for diagnosis. In chemotherapy is high (nausea and vomiting, alopecia,
AML the use of cell surface marker analysis has not bleeding / bruising, infections, anxiety and
yet altered treatment approaches. depression). Maintenance chemotherapy used in ALL
AML is the commonest acute leukemia in is generally well tolerated. Once relapse is occurred
adults (90%) whereas ALL is commonest in children second remissions are much more difficult to achieve
(90%), 70% are CALLA positive. Presentation of all particularly in AML. If the patient has an HLA-
forms of leukemia is distressingly similar, a previously identical sibling then bone marrow transplantation may
healthy person develops symptoms from reduction in be a viable alternative for AML (first remission) or
circulating blood cells (anemia, thrombocytopenia, ALL (second remission)
fever or lymphadenopathy / splenomegaly, with the
duration of illness usually being very short (a few days
to a few months). ALL may cause enlarged lymph
Page 48 of 72
SUPPORTIVE CARE granulocytic series, often back to myeloblast state (but
At presentation these patients are often the "hiatus" between blast cells and mature cells as
anemic, infected and bleeding; during remission seen in acute leukemia is absent in CGL) Anemia is
induction these potential complications are very usually mild, and a wide range of platelet counts may
common, and the following guidelines generally apply be seen (to 1000 x 109/L)
to their management: The bone marrow also demonstrates the
a. Anemia increased and left-shifted granulopoiesis, but is
- packed red cell transfusions are given as generally not required for diagnosis (there are no
needed for symptomatic anemia diagnostic morphologic features which differentiate
b. Thrombocytopenia CGL from reactive granulocyte hyperplasia) It may,
- during remission induction, spontaneous however, provide a sample for karyotyping for
hemorrhage is common if the thrombocytopenia is Philadelphia chromosome if needed. The Philadelphia
severe (platelet count 10 x 109/L) so many centers chromosome (loss of short arm of chromosome
provide prophylactic platelet transfusions during number 22 due to 22:9 translocation) is present in 85-
this period; others are more selective, attempting 90% of CGL patients; if absent, in a patient who
to monitor the seriousness of the appears to have CGL by other tests, the prognosis is
thrombocytopenia by clinical assessment (amount worsened. The Philadelphia chromosome is only rarely
of purpura, epistaxis, mucosal bleeding, etc). The present in other myeloproliferative diseases.
goal is to prevent intracerebral hemorrhage which Another test which might help differentiate
is rapidly fatal if it occurs. Any drugs which cause between CGL and leukemoid reaction (granulocytic
platelet dysfunction must be omitted e.g. Aspirin, hyperplasia) is leukocyte alkaline phosphatase
and intramuscular injections are prohibited. (LAP); this is a histochemical test for alkaline
c. Granulocytopenia phosphatase in leukocyte (neutrophil) granules – it is
- although reverse isolation procedures do not alter characteristically decreased – absent in CGL but
the frequency of serious infections in these normal – increased in leukemoid reactions. It is
patients, most acute leukemias are performed on peripheral blood, rather than bone
provided with a private room, whenver possible, marrow.
and visitors with infections are discouraged. Masks
may help if attending staff have a potentially MANAGEMENT OF CML
transmittable upper respiratory infection and hands 1. Oral chemotherapy:
should be washed before examining the patient. During chronic phase, hematological remission is
If a neuropenic patient develops a chill and/or produced in more than 70% of patients treated
fever, the patient should be promptly assessed for with Hydroxyurea or Busulphan. Hydroxyurea is
site of infection, cultured immediately (Throat preferred over Busulphan because of better
swab, urine C & S, blood cultures etc) and if a toxicity profile, progression free survival and
septicemia is likely, should be placed on broad overall survival. However, no significant
spectrum (eg. Cephalosporin plus aminoglycoside) cytogenetic remission occurs with Hydroxyurea or
intravenous antibiotics as soon as cultures have Busulphan and over all pace of progression to blast
been obtained. Granulocyte transfusions have not crisis is unchanged. These treatments are therefore
been proven to be of help in this situations. considered to be palliative.
d. Emotional support 2. Allogeneic bone marrow transplantation:
- These patients require regular visits, frank Allogeneic hematopoietic cell transplantation is
discussions and frequent reassurance from the only known curative treatment for CML.
compassionate and knowledgeable medical, However only minority of patients have HLA
nursing and other personnel to cope with the matched siblings. Treatment related morbidity and
multiple physical and emotional problems mortality is significant and increase with
which routinely are encountered. increasing age of the patient. Therefore currently
this modality is offered to patients less than 40
CHRONIC GRANULOCYTIC LEUKEMIA years of age. In younger patients HLA matched
Chronic granulocytic leukemia (CGL) unrelated donor transplantation is feasible.
accounts for 20% of all cases of leukemias, and mainly 3. Interferon Alpha
onset is in middle age although cases in both Before the use of Imatnib mesylate, interferon
adolescents and elderly patients are seen. The onset is alpha was the standard therapy for CML in patients
insidious over several months, or more, and patients not eligible for allogenic bone marrow
often present with significant splenomegaly, possibly transplantation. Interferon alpha alone or in
symptomatic and hypermetabolic symptoms (fever, combination with Cytosinarabinoside produced
night sweats, etc) Diagnosis is usually obvious after hematological remission in most patients and
clinical examination and assessment of the WBC cytogenetic responses in 20% patients. There is
differential. Leukocytosis is present with left shift in
Page 49 of 72
significant toxicity and interferon has even characteristic antigenic profile in CLL lymphocytes.
superseded by Imatnib The lymphocytes are positive for CD45, 19,20,22,23,
4. Imatinib Mesylate HLADR and characteristically show co-expression
Imatinib (Gleevec) is a tyrosine kinase inhibitor. It with T cell antigen CD5 and light chain restriction. The
inhibits PDGF, Abl & C-kit tyrosine kinases by immunophenotyping helps to distinguish it from other
interfering with the ATP binding site. It produces lymphoproliferative disorders.
hematological responses in 94% patients and Anemia and /or thrombocytopenia may result
major cytogenetic responses in 83% patients. The from marrow involvement, hypersplenism, or
drug has excellent toxicity profile with common autoantibodies (immune hemolytic anemia of IgG type
side effects being mild nausea, myalgia, edema of secondary I.T.P) Hypogammaglobulinemia is
and diarrhea. However, it is not clear at present common.
whether it is a curative modality for CML. More
potent tyrosine kinase inhibitors are currently MANAGEMENT OF CLL
undergoing clinical trials. Patients with early stage of CLL do not require
treatment. In young patients, Fludarabine +
CHRONIC LYMPHOCYTIC LEUKEMIA Cyclophosphamide is currently the treatment of
Chronic lymphocytic leukemia (CLL) is the choice. It produces higher incidence of remission
commonest form of leukemia, appearing mainly in old (complete and overall) compared to Chlorambucil or
age, often symptomless and requiring no treatment. combination chemotherapy. The drug may be
Hypermetabolic symptoms such as fever, night sweats, associated with cumulative myelotoxicity and
fatigue and weight loss may occur. Lymph nodes and increased risk of opportunistic infection.
spleen are regularly enlarged often for many years,
without causing symptoms, but can achieve such In older patients, intermittent Chlorambucil is an
enlargement as to require treatment for pressure or appropriate therapy, because of it ease of
cosmetic symptoms. administration and relatively favourable toxicity
The laboratory diagnosis is usually simple. profile.
There is lymphocytosis which may reach extreme Moncolonal antibody, anti CD20 (Rituximab) is
degrees (500 x 109/L or more) without other obvious effective when used in combination with
causes (eg. Viral infection) The lymphocytes seen are chemotherapy and anti CD52- Alemtuzumab
usually small lymphocytes and appear morphologically (Campath) have significant activity in CLL. These
normal. Frequently they are fragile so that they rupture agents are however used for patients who have failed
during preparation of the slide; they are then called Fludarabine therapy.
"smudge cells". Although not diagnostic of C.L.L they Coomb's positive autoimmune hemolytic anemia and
are so frequently seen in this type of leukemia that they immune thrombocytopnenic purpura are treated with
are quite typical. The bone marrow contains a Prednisolone.
lymphocytic infiltration but often is not needed to Radiotherapy is reserved for symptomatic lymph
make the diagnosis of CLL. Flow cytometry reveals a node enlargement not responding to chemotherapy.

Page 50 of 72
PROBLEMS FOR TUTORIALS
Questions:
1. ADULT MAN WITH LYMPHOCYTOSIS 1. What are the B-symptoms and how do you define
A 65-years-old man was brought to the emergency unit them?
by his son because he has fainted at home. According 2. What is your differential diagnosis?
to his son, the man was gradually feeling week with 3. What initial tests would you like to order?
inability to do his daily routine. He had loss of appetite 4. How do reach a final diagnosis? What do the cells
and lost 5 Kg. over the last 6 months. He had no shown below imply?
previous history of cardiac disease or hypertension but
had diabetes for which he was on oral hypoglycemic
drugs for the past 5 years.
3. POLYCYTHEMIA
Problem: A 60 year old man with history of
heavy smoking and chronic bronchitis was admitted to
On examination, the man was conscious, oriented, a
hospital with the onset of severe dyspnea. This man's
febrile, pulse: 90 b. / min. B.P: 130 / 80, he was not
chronic chest disease has been present for twenty
jaundiced but had pale conjunctiva. Examination of the
years. At the time of admission to hospital, he was
neck revealed bilateral cervical and supraclavicular
dyspneic and plethoric with both central and peripheral
lymphadenopathy. There were also two axillaries and
cyanosis. There was no clubbing. Examination of the
one inguinal lymph node. Examination of the heart and
chest revealed diffuse wheezing.
chest was normal. Abdominal examination revealed a
Laboratory Investigations:
splenomegaly of 10 cm. below the left costal margin
Hemoglobin 210 g/L
(LCM)
Hematocrit 0.69
WBC 19 x 109/L
Investigations showed:
Platelet Count 950 x 109/L
Hb. 95 g/L N M: 140-160
Chest X-ray showed over inflation of the lungs
F: 120-160
bilaterally
MCV: 105 fl (N 80- 95)
Required questions:
WBC count: 50 x 109/L Lymphocytes 80%
Platelets 90 x 109 / L 1. Is this likely to be a primary or secondary
Blood Sugar: 8.5 mmol / L Na: 140 mmol/L erythrocytosis (polycythemia)? Why?
K: 4.2 mmol/L 2. What additional investigations would you
perform?
Questions: 3. Discuss treatment in terms of Polycythemia
1. What are the important findings in the history and
clinical examination of this patient? 4. ATYPICAL LYMPHOCYTOSIS
2. What is the differential diagnosis Problem: This 18 year old woman saw her
3. What is the most likely diagnosis? physician for increasing tiredness over the past two
4. What do you think is the cause of the anemia? weeks associated with a sore throat and fever.
Physical examination revealed a young woman
who had bilateral slightly tender anterior cervical
nodes 2.5 cm in diameter. The spleen was palpable 3
cm below the left costal margin
Laboratory Investigations:
Hemoglobin 132 g/L
WBC 12.4 x 109/L
.01 bands
.46 segmented neutrophils
2. A YOUNG FEMALE WITH NECK LUMP .26 lymphocytes
A 17-year-old Kuwaiti girl sought medical at tension .25 atypical lymphocytes
because she noticed lumps in the left side of her neck. .02 monocytes
She denied any B-Symptoms. Her past medical history Platelet count 110 x 109/L
is unremarkable. Required questions:
On examination, there is a 3x3-cm firm, non-tender, 1. List 2 causes of atypical lymphocytosis
freely movable nodule in the left neck just above the 2. What is the most likely cause in this patient
sternoclavicular joint, and several smaller nodes to the 3. The patient worries about having cancer. List 3
left along the clavicle. All vital signs are normal. There reasons why you think it is unlikely
is no fever or icterus. The thyroid gland is normal. The 4. The patient requests an antibiotic for new sore
chest, heart, breasts are normal. The spleen is palpable throat. You do not agree with this. Why not, and
2 cm below the left costal margin on deep inspiration. what would you do instead
Page 51 of 72
5. What are the appropriate confirmatory tests? and some gum bleeding. By this time it was
known that her neutrophil count was very low
5. LEUKOCYTOSIS AND SPLENOMEGALY (less than 0.5 x 109/L) Describe your
Problem: This 51 year old woman was management with regard to:
referred to hospital. One month prior to admission she a. Tests to investigate the fever
had sharp pains in the left upper quadrant of her b. Any treatment for the febrile illness. What
abdomen. She palpated a large mass herself, and should she avoid?
consulted her private physician who thought the mass c. Treatment of the thrombocytopenia?
was spleen. He noted that the leukocyte count was i. What blood products should be ordered?
elevated. ii. How much?
Laboratory investigations iii. Expected response
Hemoglobin 100 g/L
Platelet count 650 x109/L 7. LEUKOCYTOSIS
WBC 70 x 109/L Problem: This 78 year old man visited his
.02 blasts family physician for an annual "checkup" He was in
.03 promyelocytes good health except for mild symptoms of congestive
.18 myelocytes heart failure being treated with digoxin and
.17 metamyelocytes hydrochlorothiazide daily.
.25 bands On examination, there was no tonsillar
.20 neutrophils enlargement, lymphadenopathy or hepatosplenomegaly
.04 lymphocytes Laboratory Investigations:
.02 monocytes Hemoglobin 148 g/L
.05 eosinophils MCV 88 fL
Required Questions: MCHC 34- g/L
1. What is the most likely diagnosis? WBC 58.0 x 109/L
2. What confirmatory tests might you order .15 neutrophils
3. The patient asks if her spleen should be removed .65 lymphocytes
since it bothers her. Do you agreed and why? .20 smudge cells
4. The patient has just started a clothing business and Platelet count 175 x 109/L
wants to make plans for the future. What advise Required Questions:
would you provide regarding: 1. What is the most likely diagnosis to explain
a. Possibility of bone marrow transplantation the blood findings?
b. Side effects of the treatment you would 2. Are any further tests required
prescribe 3. What advise would you give this patient
c. Complication which is likely to occur and regarding:
when. a. Need for treatment now?
5. New treatment. b. Indications for treatment in the future
c. Side effects of treatment if required?
6. PANCYTOPENIA d. Overall disease course
Problem: This 59 year old woman was
referred to hospital because of a low WBC count. The 8. LYMPHADENOPATHY
patient denied malaise of fatigue. Two weeks before Problem: A 30-year old female who was
admission, she had an acute respiratory tract infection previously well until 4 weeks ago when she noted a
treated with penicillin. Except for pallor and few lump in her left axilla. During the past 2 weeks she has
bruises, there were no other physical findings. complained of fatigue. Physical examination revealed
Laboratory Investigations a 3 cm, non-tender soft mass in the left axilla. There
Hemoglobin 80 g/L were no abnormalities detected on breast exam and
MCV 82 fL there as no evidence of splenomegaly or
MCHC 350 g/L lymphadenopathy elsewhere.
WBC 3.8 x 109/L Laboratory Investigations:
Platelet count 15 x 109/L Hemoglobin 115
Required Questions Hematocrit .3
1. List any further laboratory tests you would White Blood Count 4.5
request Platelet count 200
2. List 2 possible diagnosis for the blood changes Chest X-ray small mediastinal mass
3. The day after admission, her temperature
spiked to 39oC and she had a chill. She also Required Questions:
developed many bruises, recurring nosebleeds
Page 52 of 72
1. List 3 additional pieces of information from 4. What other investigations are necessary if this
the history you would require is either Non-Hodgkin's Lymphoma or
2. What test would you arrange to make the Hodgkin's Disease?
diagnosis? 5. Assuming the diagnosis of Hodgkin's Disease,
3. What are the most likely diagnoses? what additional investigations may be done?
6. What is the importance of the information
gained in the above questions?

Page 53 of 72
MYELOPROLIFERATIVE DISORDERS
The designation "myeloproliferative disorder" In both of the above possible categories the
is a general term given to wide range of malignant abnormality is usually limited to increased Hb, Hct,
proliferation of myeloid tissue, which can be classified and RBC count, without an elevation of WBC count or
as follows: platelet count. The red cell mass, as assessed by Cr51
blood volume, is increased in both secondary
MYELOPROLIFERATIVE DISORDERS polycythemia, and primary polycythemia (P. vera)
1. Polycythemia vera Polycythemia vera is a malignant proliferation of
2. Myelofibrosis (myelosclerosis) myeloid cells usually involving all marrow cell lines
3. Myeloid metaplasia (with or without leading to erythrocytosis, leukocytosis (neutrophilia)
myelofibrosis) and thrombocytosis. Splenomegaly is present in 75%
4. Chronic granulocytic leukemia of patients at diagnosis)
5. Thrombocythemia Common symptoms include plethora (ruddy
6. Acute myeloblastic leukemia cyanosis of face and extremities), hyperviscosity
We have included CGL and AML under symptoms (transient bleeding, TIA's, angina,
leukemias, and will restrict further discussions to claudication etc) bleeding and pruritus (which is
polycythemia, myelofibrosis (myeloid metaplasia) common, often sever, and aggravated by bathing "bath
and thrombocythemia. pruritus")
Management includes control of red cell mass
POLYCYTHEMIA VERA (hyperviscosity) and prevention of bleeding
Before deciding whether a patient has complications. If erythrocytosis is marked but not
polycythemia vera, the physician must first usually leukocytosis or thrombocytosis, then a therapeutic
exclude two other possibilities: phlebotomy program (removal of 500 ml blood usually
b. Spurious (stress, or relative) erythrocytosis – weekly) may be instituted; usually this must be
patients with this disorder are detected by the continued till the patient develops iron deficiency,
isolated findings of elevated Hb and Hct, often in limiting erythropoiesis and this may take several
association with hypertension (Gias-bock's months. If there is complicating thrombocytosis,
syndrome) Blood volume studies (Cr51 labelled symptomatic splenomegaly or hypermetabolic
RBC's) demonstrate a normal red cell mass, symptoms, then radioactive phosphorus (P32) will
however, and the "apparent" erythrocytosis is due provide good long term control of P.vera, and should
to a reduction in plasma volume. This condition be considered particularly in older patients.
requires recognition, to differentiate it from true Chemotherapy has also been used, but there is a
polycythemia, but no treatment. definite leukemogenia potential with either
chemotherapy (chlorambucil, myleran) or P32 so this
c. Secondary polycythemia (erythrocytosis)- this must be carefully assessed for each patient. Younger
represents a response to tissue hypoxia, or patients should receive hydroxyurea, as chemotherapy,
autologous erythropoietin production and the due to its lower leukemogenic risk. Recently Aspirin
following should be considered: has been shown to decrease the risk of thrombosis.
i. Chronic lung disease – particularly chronic
bronchitis Uric acid is usually increased and most
ii. Congenital heart disease – cyanotic, with right to patients are treated with allopurinol to prevent gout or
left shunt other complications of hyperuricemia.
iii. Renal disease – renal cell carcinoma, cysts, renal
artery stenosis MYELOFIBROSIS, MYELOID METAPLASIA
iv. Liver disease – hepatoma These two conditions often exist together,
v. High altitude although myeloid metaplasia can occur without
vi. Hemoglobinopathy – high affinity hemoglobin fibrosis in the bone marrow (called "agnogenia"
(rare) myeloid metaplasia). The marrow fibrosis may result
vii. Cerebellar hemangioblastoma (rare) in bone proliferation as well (myelosclerosis) and be
visible on x-ray but usually it doesn't. It does result in
Most of the above can be ruled out on clinical decreased hematopoiesis, and the concomitant myeloid
assessment. The one diagnosis which must not be metaplasia present mainly in the liver and spleen is
missed is the renal cell carcinoma which may still be rather inefficient. Accordingly, the patients with these
curable if detected at a localized stage; if any doubt disorders usually have anemia and thrombocytopenia,
exists as to a cause for erythrocytosis, appropriate both of which may be severe, whereas the leukocytes
investigations of the urinary tract should be are often increased and left shifted.
considered.
Page 54 of 72
The peripheral blood film is usually be helpful if deficiency is documented. Splenectomy
characteristic, with a "leukoerythroblastic" blood may be required to control symptoms of massive
picture – nucleated RBC's and left-shifted granulocyte splenomegaly; this operation is risky, however, and
series. Other features include teardrop poikilocytes, platelet function should be studied prior to surgery
and megathrombocytes – all of these abnormalities since many patients have abnormally functioning
result from abnormal hematopoiesis taking place platelets in addition to the thrombocytopenia.
outside the bone marrow. Marrow biopsy is required to
demonstrate the marrow fibrosis. ESSENTIAL THROMBOCYTHEMIA
Patients usually present with symptoms of If the thrombocytosis is extreme (1,000 x
anemia and/or thrombocytopenia, with systemic 109/L) it should probably be controlled by
complains of fever, night sweats, and weight loss. chemotherapy Hydroxyurea (myleran) particularly if
Splenomegaly is usually prominent and may be any surgery is planned and is elective enough to
massive producing major pressure symptoms. Splenic provide time for this treatment to be effective (1-2
infarcts are common. months) Emergency treatment with plateletphoresis is
Treatment is unsatisfactory – Supportive care rarely needed. These patients tend to develop other
with red cell transfusions for symptomatic anemia and myeloproliferative disorders (polycythemia, leukemia)
platelet transfusions for thrombocytopenia bleeding but usually many years pass before this occurs. Baby
complications may be required. Androgens may Aspirin may reduce the risk of arterial thrombosis.
provide mild benefit for the anemia and folic acid may

Page 55 of 72
MODULE III & IV
HAEMOSTASIS & THROMBOSIS
OBJECTIVES
• Discuss common causes of bleeding disorders and their pathogenesis.

• Describe the clinical presentation, plan of investigations and interpret, integrate and
correlate salient points in clinical history, physical findings and laboratory data so as to
reach a correct diagnosis in a patient presenting with a bleeding disorder.

• Describe the common mode of clinical presentation and demonstrate ability to interpret
relevant laboratory data in patients with following bleeding disorders.

• Haemophilia

• von Willebrand's disease

• Acquired coagulation deficiency secondary to vitamin K deficiency, liver disease, chronic


renal failure, and DIC.

• Discuss common causes of thrombocytopenia.

• Plan and interpret salient points in clinical history, physical findings and relevant
laboratory data to establish diagnosis of the type and cause of thrombocytopenia in a
patient.

• Plan and administer rational therapy to a patient with thrombocytopenia.

• Describe the indications, schedule of administration and side effects of the use of fresh
frozen plasma, cryoprecipitate and platelets concentrates in the management of bleeding
disorders.

• Discuss pathophysiologic mechanisms of thrombosis and describe molecular basis of


inherited thrombophilia.

• Enumerate common causes of acquired and inherited thrombotic disorders.

• Demonstrate ability to correlate salient points in clinical history and physical findings and
to plan and interpret laboratory investigations so as to make dignosis of a thromboembilic
disorder.

• Describe mechanism of action, dose, frequency, mode of administration, and adverse


effects of heparin and coumadin anticoagulants

Page 56 of 72
BLEEDING DISORDERS
GENERAL OBJECTIVES 1. Defective platelet function
The student should have a general a. Hereditary – von Willebrand's Disease
understanding of hemostasis, the types of common or (VWD), defective aggregation
important bleeding problems which occur with b. Acquired – medications, anemia, myelo-
hemostatic defects, and the principles of management proliferative diseases.
for these problems. 2. Vascular Disorders
a. Allergic (Henoch-Schonlein purpura)
SPECIFIC OBJECTIVES b. Non-allergic – mechanical, senile, steroid,
At the end of this section, the student should dysproteinemia, connective tissue disorders.
be able to:
1. list possible causes of bleeding problem given APPROACH TO PATIENTS WITH BRUISING
either a clinical presentation of bleeding, abnormal (PURPURA)
tests of hemostasis or both In general, patients with thrombocytopenia,
2. List any additional tests required, if any to define defective platelet function, or vascular disorders tend
the precise cause of a bleeding problem. to bruise either with minimal trauma or even
3. Describe the clinical presentation and expected spontaneously, depending on the severity of the
laboratory abnormalities for common or important problem. The various factors tend to be additive ( i.e.
bleeding problems; these include one would expect more bruising in an elderly patient
Hemophilia, von Willebrand's disease, with defective connective tissues, who is
Acquired coagulation deficiencies, secondary thrombocytopenia and taking Aspirin. Also the
to vitamin K deficiency, liver disease and DIC, combination of platelet/vascular disorders discussed in
thrombocytopenia secondary to ITP, this section with coagulation disorder discussed in
consumption or decreased production. subsequent sections usually leads to more severe
4. Demonstrate an understanding of the use of bruising and bleeding problems than either would by
heparin and coumarin anticoagulants including itself.
types of drugs, dosage, frequency and route of The commonest manifestations of these
administration for both prophylaxis and therapy of disorders are bruises and petechiae involving the
thromboembolic disease muco-cutaneous surfaces. Petechiae appear as small (1
5. Describe the mode of inheritance for hereditary – 3 mm diameter) red spots, which do not blanch; they
coagulation disorders may occur on any cutaneous or mucosal surface but
6. Describe the use of Fresh Frozen Plasma, tend to be more evident on dependent areas, such as
Cryoprecipitate and Platelet concentrates in the legs in ambulatory patients. Widespread and/or
management of bleeding disorders, including mucosal lesions indicate a serious problem.
indications and contradictions for its use, expected In addition to these manifestations patients
response and possible complications. with several platelet and vascular disorders may also
present with abnormal bleeding at many sites (e.g.
PLATELET / VASCULAR DISORDERS OF epistaxis, GI hemorrhage, cerebral hemorrhage,
HEMOSTASIS intramascular hematomas, etc). Excessive bleeding
often occurs with surgery and trauma
CLASSIFICATION OF PURPURA (BRUISING)
CLINICAL ASSESSMENT
THROMBOCYTOPENIC PURPURA Special attention must be paid during the
1. Decreased Production interview to elicit information about current bleeding
a. Hereditary – very rare and bruising problems as well as previous history,
b. Marrow suppression – medications, radiation, including surgery, dental extractions and childbirth. A
alcohol, chemotherapy meticulous enquiry into the use to drugs, medications
c. Marrow infiltration – leukemia, metastatic and alcohol is mandatory and the review of the family
tumors history must be thorough to detect hereditary
d. Marrow ineffective – megaloblastic, problems. 70% of all hereditary bleeding diseases will
dysmegakaryopoiesis have an evident family pedigree
2. Decreased Survival Physical examination should document the
a. Immune – I.T.P, medications, sepsis, neonatal extent and location of petechiae and bruises as well as
3. Dilutional the presence of absence of other vascular
a. Massive transfusion malformations e.g. telangiectasia, hematoma, confluent
hemorrhagic stomatitis. It is also important to rule out
NON-THROMBOCYTOPENIC PURPURA
Page 57 of 72
other underlying diseases such as liver disease or In thrombocytopenic patients a bleeding time
hematologic disorders. is usually contraindicated unless the result would
have a direct influence on management.
LABORATORY ASSESSMENT Limitation: This test is not precise and
1. CBC, Differential, Platelet count: does not have predictive value.
Platelet must be either estimated or counted in
a bleeding or bruising patient to document 4. Bone marrow:
thrombocytopenia. If the estimate made on A bone marrow aspirate and preferably a
observation of the blood film is normal, this is biopsy will quickly differentiate between
sufficient to rule out thrombocytopenia. If platelets thrombocytopenia due to decreased production
appear reduced, a platelet count is usually (decreased megakaryocytes) versus that due to
performed to quantitate platlet number accurately. increased destruction (normal to increased
Mordern cell-counter equipment provide accurate megakaryocytes) it is usually performed when the
counts rapidly, often alont with other cause for thrombocytopenia is not obvious by
measurements (Hb, WBC, indices, etc); if this other non-invasive tests, or clinical examination.
equipment is not available then manual platelet
counts must be performed – these are more time SIGNIFICANCE OF THROMBOCYTOPENIA
consuming and less accurate. When AND ASSOCIATED CLINICAL FINDINGS
thrombocytopenia is sever (<109/L) most
laboratories report it as such rather than providing Severe thrombocytopenia
a figure since counts below that level cannot be (platelet count < 20 x 109/L and particularly < 10
accurately measured. x109/L) – risk of spontaneous bruising and
All Abnormal platelet counts must be confirmed bleeding, which may be life threatening (eg.
by microscopic examination of a blood film. Intracranial hemorrhage)
The CBC and Differential are required to
determine whether a thrombocytopenia is selective Moderate thrombocytopenia
(usually indicating peripheral destruction, often (platelet count 20 – 80 x 109/L – may bruise with
immune in origin) or associated with decrease in mild trauma but usually not spontaneously;
other cellular elements (anemia, leucopenia) excessive bleeding is likely to occur at surgery.
and/or an abnormal differential (eg leukemia).
Morphology of the platelets is important to assess Mild thrombocytopenia
eg. Abnormal shape or cytology (platelet count 80 – 150 x 109/L) – no problem
2. Coagulation Screening Tests: expected with or without trauma or surgery.
If abnormal in a thrombocytopenic patient this
suggest involvement with coagulation system. NOTE:
Certain diagnosis, such as D.I.C, liver disease / 1. Ingestion of drugs which affect platelet function
hypersplenism and lupus require exclusion. will make any degree of thrombocytopenia more
3. Bleeding Time: likely to cause bleeding/bruising and should be
This test is usually performed by making 2 avoided.
short standard incisions on the patient's forearm, 2. Intramuscular injections should be avoided in all
often using a template to regulate the length and patients with platelet /vascular and coagulation
depth of the incisions. The time taken for the disorders to avoid development of intramuscular
bleeding from the incisions to cease is a good hematomas.
measure of the in vivo significance of 3. Both the cause and the degree of
thrombocytopenia or platelet dysfunction. thrombocytopenia must be considered when
It is usually performed in a bruising patient assessing the clinical significance of
with normal numbers of platelets to detect platelet thrombocytopenia (patients who are compensating
function abnormalities. If it is normal, and platelet for increased platelet destruction such as I.T.P are
number is normal then no further platelet tests (eg much likely to bleed than those suffering from
aggregation) are indicated. A prolonged bleeding decreased platelet production such as patients with
time in the presence of normal platelet number leukemia and aplastic anemia.)
implies a platelet function defect and further The role of platelet transfusions in the
studies may be indicated unless the cause is management of patients with thrombocytopenia or
evident (eg. Uremia) One should exclude the platelet dysfunction is outlined in a previous
recent ingestion of drugs which alter platelet section (Blood Transfusion Practice).
function (note that ASA may prolong the bleeding
time for up to 1 week) prior to ordering the test. COMMON AND/OR IMPORTANT DISORDERS
Generally speaking the most common causes
of bleeding under the heading of “Platelet and Vascular
Page 58 of 72
Disorders" are thrombocytopenias. The most common
thrombocytopenias are probably: PLATELET FUNCTION DEFECTS
1. Those seen with general marrow suppression or VWD is one of the most common of the
aplasia produced whether accidentally (i.e. hereditary hemostatic disorders. Other hereditary
Chloramphenicol, Phenylbutazone) or platelet function disorders are rare.
therapeutically (i.e. Myelosuppressive drugs) The acquired platelet defect seen with ASA is
2. Those secondary to marrow infiltrations such as important as it is so common, and may aggravate other
leukemia and secondary tumor. hemostatic problems. The platelet defects seen in other
3. Hypersplenism diseases are usually mild, but may cause serious
4. In association with D.I.C problems at surgery.
5. The specific immune thrombocytopenias (ITP or Treatment of vWD requires the injection of
following certain drugs such as Quinine and DDAVP (arginine vasopressin) to release vWF from
Quinidine, Heparin) the individual's endothelium or viral attenuated
6. Association with infections, especially HIV products that contain vWF (eg. Humate-P)
Other than the thrombocytopenias the platelet
function defect in association with Von Willebrand's VASCULAR DISORDERS
disease and with aspirin are important to be aware of Allergic (Henoch-Scholein Purpura):
as are several of the vascular disorders such as the non- This disease is seen not uncommonly in
allergic, non-thrombocytopenic simple purpuras seen children, especially boys with peak incidence at 3- 7
commonly in elderly and in patients on steroids, and years and usually occurring 1-3 weeks following an
allergic (Henoch-Schonlein) purpura seen not upper respiratory infection. Onset is sudden with
uncommonly in children. malaise, headache, fever, urticarial rash, and 50% have
The following are some comments regarding some joint or abdominal pain. There are no specific lab
of the above subjects which are not covered elsewhere: findings.
Treatment with corticosteroids is usually not
IMMUNE THROMBOCYTOPENIAS effective but commonly used in the sicker child with
Acute I.T.P joint and abdominal pain. The problem usually last 2-4
This disease occurs predominantly in children weeks but relapses are not uncommon.
of either sex consisting usually of a severe abruptly Non-Allergic:
onsetting thrombocytopenic purpura usually 2-21 days Non-thrombocytopenic purpuras such as
following viral infection. Most recover spontaneously simple purpura (seen in 50% of healthy women) senile
in 1-2 months. It is important to rule out bacterial purpura and steroid purpura are common but usually
sepsis as a cause. very mild and require no treatment.
Treatment with Prednisone 1-2 mg./kg or
intravenous gamma globulin 1g/kg is often given if THROMBOCYTOSIS
significant mucocutaneous hemorrhage is present. 80% Thrombocytosis applies to any patient with a
will recover regardless of treatment. 50% will have platelet count greater than normal (450 x 109/L). Mild
normal platelets in 6 weeks and 80% are well in 6 degrees are fairly common and usually not clinically
months. Approximately 20% become chronic lasting 6 significant. Severe degrees (1,000 x 109/L) may result
months or more and some remain severe enough to in serious problems (eg CVA) particularly in patients
require steroids or other immunosuppressive therapy with underlying vascular disease or stasis. Therefore,
for 6-12 months with splenectomy only being the management of these patients requires knowledge
performed if no other therapies prevent significant of the platelet count, the cause of the increased
bleeding. platelets, and the general medical/surgical condition of
the patient.
Chronic I.T.P:
This is a disease of adults occurring in 3:1 in CLASSIFICATION
females, usually onsetting insidiously and rarely 1. Primary:
resolve spontaneously. a. Myeloproliferative disorders
Usual treatment is Prednisone 1 – 2 mg/kg b. Essential thrombocytosis
with a response in the platelet count usually occurring (thrombocythemia)
in 3-4 days to 2 weeks. Splenectomy is considered if c. Associated with other myeloproliferative
no response occurs after several weeks of large doses disorders, eg. Myeloid metaplasia /
or in patients continuing to require Prednisone for myelofibrosis, polycythemia, rubra vera,
longer than 6 months or having major side effects and chronic myelogenous leukemia
continuing bruising and bleeding occurs. Because of 2. Secondary:
the extremely short life span of transfused platelets, a. Post-splenectomy
platelet transfusions are used only in life-threatening b. Response to chronic bleeding
hemorrhage. c. Iron deficiency
Page 59 of 72
d. Chronic Inflammation / infection search to detect what is sometimes a small primary
e. Underlying malignancy neoplasm. Patients with secondary thrombocytosis
rarely require treatment for the thrombocytosis itself
The diagnosis and management of patients since it is usually mild (< 1000 x10 9/L),
with primary thrombocytosis is discussed in the uncomplicated and transient or reversible with
myeloproliferative section of the Lymphadenopathy treatment of the underlying disease. Drugs that alter
and Splenomegaly Section. platelet function (e.g ASA) may be used if
In patients with secondary thrombocytosis the thrombocytosis is more sever, or occurs in a patient at
cause may be obvious (post-splenectomy, iron high risk for vascular occlusions but this is seldom
deficiency) but in others it may be very obscure (e.g. required.
underlying malignancy) and necessitate a thorough

Page 60 of 72
COAGULATION DISORDERS
Coagulation disorders comprise a wide variety increasing the reliability and reproductivity. The
of both hereditary and acquired hemostatic problems. normal range varies from one laboratory to
The hereditary disorders vary from uncommon to very another, but usually the upper limit is less than 40
rare, and only the more common types will be seconds; it will be prolonged when the amount of
discussed in some detail; the general principles, any single factor decreases to less than
however, apply to them all. The acquired disorders are approximately 30% ( 0.30 u/ml)
more common, usually being secondary to other The test measures the intrinsic clotting
conditions (infections, pregnancy, liver failure), system as mentioned previously, defects in the
medications (Coumadin) or poor nutrition. common pathway (Factors X, V, II and I) prolong
the P.T.T. as well as the INR; isolated deficiencies
APPROACH TO PATIENTS WITH BLEEDING in factors XII, XI, IX, and VIII result in
PROBLEMS prolongation of P.T.T. only)
Whereas patients with platelet / vascular
disorders predominantly with excessive bleeding 3. Thrombin Time (T.T) or Thrombin Clotting
involving the mucocutaneous surfaces, the Time (T.C.T)
coagulopathies present mainly with bleeding of The thrombin time measures the brief time
potential spaces eg. Joints or soft tissue eg muscles as (about 9 seconds) required to convert fibrinogen to
opposed to bruising, although there is some overlap. fibrin after the addition of thrombin to citrated
With minor surgery, mild trauma, and dental plasma. It is most often prolonged when there is a
extractions, bleeding is usually prompt with platelet deficiency of fibrinogen, a defect in fibrinogen
disorders but may be delayed with coagulation defects (dysfibrinogenemia) or if heparin is present in the
since initial hemostasis is provided by platelets samples.
(platelet plug) and backed up later by a fibrin 4. Fibrinogen (Factor I)
thrombus. The fibrinogen level can be measured in
In coagulation disorders bleeding may occur plasma; since it is an “acute phase reactant” it is
from any orifice and in almost any organ. Typically, in frequently elevated in many infections,
hereditary coagulation factor deficiencies, inflammatory or traumatic illnesses. A decreased
hemarthrosis are most frequent and disabling. fibrinogen level may be isolated as a rare
During the clinical examination of patients hereditary deficiency, but more commonly is
with bleeding problems careful attention must be paid reduced along with other factors in liver disease,
to past history and family history. The pattern of D.I.C or fibrinolysis.
inheritance will be helpful (e.g. males affected and 5. Fibrin-Fibrinogen Degradation Products (FDP)
females as carriers of sex linked recessive for FDP are the result of fibrin and fibrinogen
hemophilia, but equal distribution in males and proteolysis, indicating excess plasmin activation.
females for VWD) They do not differentiate between primary and
secondary fibrinolysis and mild to moderate
LABORATORY TESTS FOR COAGULATION elevations are commonly seen in seriously ill
DISORDERS patients or physiologically stressed individuals.
The end result of most coagulation test is the Marked elevations, particularly when associated
formation of a fibrin clot which is usually detected by with other coagulation deficiencies and
a variety of automated equipment. The main tests are: thrombocytopenia, require that D.I.C be ruled out.
Elevated FDP are also seen in severe liver disease,
1. International Normalized Ration (INR) since plasmin is not metabolized properly in the
(Prothrombin Time (P.T.)) damaged liver. Since other coagulation
This test measures the extrinsic clotting system deficiencies and thrombocytopenia may be seen in
by adding extrinsic thromboplastin to citrated liver disease, as well as D.I.C, it is extremely
plasma. Prolongation greater than 1.2 is considered difficult to differentiate between these two
abnormal. An isolated deficiency of factors VII, X, problems. A test for fibrin monomers (protamine
V, II or I will prolong the INR (note that sulphate test) may help since it should not be
deficiencies of the common pathway usually positive if intravascular thrombin activation has
prolong both INR and PTT; isolated factor VII occurred e.g. deposition of fibrin monomer and
deficiency will only prolong the INR) polymer.
2. Partial Thromboplastin Time (P.T.T): More recently, a test called D. Dimer has
This is also known as activated P.T.T since been developed. These tests measures degradation
current tests incorporate some form of activator products of cross linked fibrin and thus not
Page 61 of 72
fibrinogen breakdown products. This test is
replacing FDP measurement in many laboratories HEMOPHILIA B DISEASE (FACTOR IX
as it does not require special collection tubes. DEFICIENCY)
This deficiency is less common than
HEREDITARY COAGULATION DEFICIENCIES Hemophilia A but has the same inheritance and clinical
Hereditary deficiencies of each clotting factor presentations depending on the severity of the
have been reported. Most of these deficiencies are very deficiency. It can only be differentiated by coagulation
rare; only deficiencies of Factor VIII (Hemophilia, and factor assays which are only reliably performed in
VWD), Factor IX (Christmas Disease) and Factor XII laboratories experienced with these tests.
(Hageman deficiency) warrant more discussion. In The same principle apply to the management
screening for all hereditary deficiencies, it is important of factor IX deficiency as for factor VIII deficiency
to remember what the INR, PTT tests measure at their regarding the need for surgical prophylaxis, and
various levels of sensitivities. adjusting type and frequency of coagulation factor
support depending on severity of deficiency and the
Deficiencies of Factors XII, XI, IX, VIII clinical circumstances. If the deficiency is mild to
→ Prolonged PTT, Normal INR moderate it should be corrected with plasma ( usually
Deficiency of Factor VII FFP although stored plasma would suffice), but if
→ Prolonged INR, Normal PTT severe then adequate replacement would require
Deficiency of Factors X, V, II, I infusion of a commercial Factor IX concentrate (again
→ Prolonged INR and PTT a pasteurized pooled lyophilized product with
decreased risk of viral borne disease transmission).
CLASSICAL HEMOPHILIA (HEMOPHILIA A)
(Factor VIII Deficiency) von WILLEBRAND’S DISEASE (VWD)
This is the commonest hereditary factor vWD is inherited as an autosomal dominant
deficiency. It is inherited as a sex-linked recessive disorder affected both males and females equally. In
disorder, being manifest almost entirely in males, with addition to Factor VIII deficiency, which is usually
females as carriers. 25% of patients have no family mild, the lack of von Willebrand factor also causes a
history. platelet dysfunction. Measurement of Factor VIII
The clinical problems vary with severity of the coagulant and VWF antigen, VWF Ristocetin cofactor
deficiency. Mild hemophiliacs (5-30%) will only have are necessary for making a diagnosis. However they
excessive bleeding with surgery or trauma, whereas at carry a risk of thrombotic complications. Pure factor
the other end of the spectrum the severe hemophiliacs IX also available.
(< 1% Factor VIII) are constantly threatened with
spontaneous hemorrhagic problems (mainly The preferred treatment for such cases is
hemarthrosis, intramuscular hematomas, and DDAVP. A cryoprecipitate is rich in vWF and is
hematuria) generally administered in a dose of one bag per 10 kg
All hemophiliacs must avoid injury, and body weight. Certain virus-attenuated concentrates
refrain from any medications (eg. ASA) which would (Humate-P) contain substantial amounts of vWF and
alter platelet function and enhance the bleeding are preferred to cryoprecipitate if available.
problem. Patients with mild hemophilia or who are
carriers may only require DDAVP to temporarily boost
the individual’s endogenous Factor VIII for minor HAGEMAN FACTOR DEFICIENCY (FACTOR
surgical procedures and then it should be given prior to XII)
and post operatively until wound healing has occurred. This deficiency is mentioned only to point out
Moderate and severe hemophiliacs on the other hand, that it may cause a prolonged P.T.T. and therefore must
will require regular infusions of Factor VIII be differentiated from other factor deficiencies (VIII,
concentrate at onset of joint pain, or on a prophylactic IX, and XI) by appropriate Factor Assays. However, it
basis, often given by themselves on a home care does not result in clinical bleeding problems since it is
program monitored by a Hemophilia Centre; the a "contract factor" and therefore causes only an "in-
predictability, efficiency and ease of administration vitro" abnormality.
outweigh the possible risks of disease transmission
such as hepatitis for these patients so commercial ACQUIRED COAGULATION DEFICIENCIES
factor VIII concentrates are favored. HIV infection is
not considered a significant contaminant since the VITAMIN K DEFICIENCY
introduction of improved pasteurized concentrates in The Vitamin K-dependent coagulation factors
1988. Further the recombinant factor IV is generated are Factors II, VII, IX and X. If the factor deficiency is
through DNA technology. This makes it save from mild only the INR is elevated, but if more severe then
infection point of view. However, formation of both INR and P.T.T are elevated since coagulation
antibodies directed to factor VIII is still a problem. factors of both intrinsic and extrinsic systems are
Page 62 of 72
involved. The thrombin time and fibrinogen levels are - septic abortion
normal thereby differentiating Vitamin K deficiency - amniotic fluid embolism
from either heparin effect (prolonged thrombin time - eclampsia
but normal fibrinogen) or liver disease (low fibrinogen, 2. Infections
also). Causes of Vitamin K deficiency include: - especially gram negative septicemia
1. Dietary deficiency – this is most commonly seen 3. Surgery
in ill patients on intravenous for 1-2 weeks - especially prostate surgery, and surgery on
particularly if also on antibiotics which kill the cancer patients
bacterial flora in the intestine. 4. Immediate Hemolytic Transfusion Reaction
2. New born (Hemorrhagic Disease of Newborn) – - Eg. ABO incompatibility
prevented by routine vitamin K 5. Malignancies
3. Malabsorption - mainly prostate Ca., promylelocytic leukemia
4. Chronic billiary tract obstruction As mentioned previously, the multiple hempostatic
5. Oral anticoagulants – vitamin K antagonists abnormalities of liver disease closely mimic D.I.C and
Patients who are at risk of developing Vitamin K diagnosis is very difficult in jaundiced patients. There
deficiency should receive prophylactic vitamin K, are also other conditions which present with
usually by parenteral route. fragmentation hemolysis and thrombocytopenia (eg.
Patients who have established vitamin K Thrombotic thrombocytopenic purpura or T.T.P., and
deficiency should receive parenteral vitamin K (5-10 Hemolytic-Uremic Syndrome, or HUS) where fibrin
mg I.V) but this requires up to 8 hours to correct the formation (thrombin activation) has not occurred.
deficiency and up to 24 hours for this to be complete.
If the patient is bleeding and one cannot wait for MANAGEMENT OF D.I.C INVOLVES:
complete correction of the INR by the vitamin K then 1. Aggressive treatment of underlying cause
plasma infusions can provide immediate but temporary - evacuation of uterus for obstetrical
correction; 2-4 or more units, of FFP, are usually complications
required to correct the prolonged INR/PTT. - broad spectrum I.V. antibiotics for septicemia
2. Replacement of deficient (consumed) factors and
LIVER DISEASE platelets
All coagulation factors except for factor VIII - FFP or possibly cryoprecipitate for coagulation
are produced in liver, so severe liver disease is defects
associated with a decrease in most factors. Often - Platelet concentrates for thrombocytopenia
platelets are also reduced due to hypersplenism and - Some clinicians also advocate heparin to stop
since fibrinolysis is also increased the multiple the activation of coagulation, but most do not
abnormalities (prolonged INR, P.T.T, T.T., decreased since these patients frequently bleed much
fibrinogen, platelets and increased FDP) mimic those more when heparinized. It is more likely to be
seen with D.I.C ordered for patients who have subacute or
Except in uncomplicated obstructive liver chronic D.I.C. often when the underlying
disease the coagulation defects are not corrected by cause cannot be quickly controlled (eg
parenteral vitamin K. For temporary correction, during promyelocytic leukemia, prostate cancer).
times of significant bleeding, infusions of FFP may be
required (again usually 4 or more units are needed to DILUTONAL COAGULOPATHY
completely correct the coagulopathy). If During the management of serious bleeding
thrombocytopenia is moderate to severe then platelet problems with massive transfusions (e.g. Greater than
transfusions may also be used. 50% of blood volume exchange in 24 hours) there is a
decline in coagulation factors since the replacement
DISSEMINATED INTRAVASCULAR products (packed red cells, crystalloid or albumin
COAGULATION (D.I.C) solutions) do not contain coagulation factors. If stored
This occurs when there is a generalized plasma is also used it will lack labile factors (V and
activation of the coagulation factors with the final VIII)
elaboration of thrombin turning fibrinogen to fibrin. It Therefore it is appropriate to monitor INR and
results in fragmentation of RBC's, consumption of P.T.T in massively transfused patients, or preferably to
coagulation factors and platelets, deposition of fibrin supply FFP judiciously (e.g. 1 unit FFP for each 1-2
and stimulation of secondary fibrinolysis. As a result equivalent units of stored plasma, 5% albumin or
there is anemia, hemolysis, thrombocytopenia, crystalloid) after a total blood volume exchange has
coagulation deficiencies (prolonged INR, P.T.T. , T.T., occurred in a short period to prevent the development
decreased fibrinogen), and increased FDP. of a coagulopathy. Platelets are also not present, or
CAUSES functionless, in most blood components, but significant
1. Pregnancy-related thrombocytopenia from dilution during massive
- abruptio placenta, retained placenta transfusion usually takes much longer to develop than
Page 63 of 72
does the coagulopathy. Platelets are also not present, or ­ Unexplained neonatal thrombosis
functionless, in most blood components, but significant ­ Family history of thrombosis
thrombocytopenia from dilution during massive
transfusion usually takes much longer to develop than ­ Recurrent fetal loss
does the coagulopathy. Platelets are also not present, or
functionless, in most blood components, but significant Types of thrombophilia:
thrombocytopenia from dilution during massive 1. Hereditary
transfusion usually takes much longer to develop than b. Natural anticoagulant deficiencies:
does the coagulopathy. The final amounts of ­ Activated protein C resistance (Factor V
replacement platelet and coagulation factors will be leiden) 25 – 40 %
determined by the clinical status of the patient eg. Age, ­ Protein C deficiency 5%
underlying conditions etc.
­ Protein S deficiency 5%
­ AntithrombinIIIdeficiency 5 %
Venous thrombo-embolic disorders
Deep venous thrombosis (DVT) affects mainly the
veins in the lower leg and the thigh. It involves the c. Dysfibrinolytic disorder:
formation of a clot (thrombus ) in the larger veins of ­ Dysplasminogenemia
the area. This clot may interfere with circulation, and it ­ tPA deficiency
may break off and travel through the blood stream ­ PAI dysregulation
(embolize). A resulting embolus can lodge in the ­ alpha-2 microglobulin deficiency
lungs, heart, or other area, causing severe damage to
that organ.
d. Others:
A pulmonary embolism (thromboembolism) occurs
when a blood clot, generally a venous thrombus, ­ G20210A prothrombin gene mutation
becomes dislodged from its site of formation and ­ Dysfibrinogenemia
embolizes to the arterial blood supply of one of the ­ Homocysteinemia
lungs.
2. Acquired
Pathogenesis:
Thrombosis is most often due to combinations of a. Antiphospholipid antibody syndrome
Virchow’s triad: b. Malignancy
­ Hematological : . PNH
• Problems with the vessel wall (i.e. . Acute promyelocytic leukemia
atherosclerosis) . PNH
• Problems with the blood flow (i.e. sluggish or
turbulant flow) - Oncologic: .Pancreatic
• Problems with the blood contents (i.e. low . Gastric
natural anticoagulant levels) . Mucin secreting
c. Nephrotic syndrome
Venous thrombosis is more predominantly a fibrin- d. Surgery and immobolization
dependent process involving the pro-coagulant cascade Normal ranges:
of proteins. In contrast, arterial thrombosis is more Protein C 70 – 140 %
predominantly a platelet-dependent process. These Protein S 60 – 140 %
different pathophysiologies influence the efficacy of AT-III 80 – 125 %
treatment strategies. APTT/APC < 2.2 males
APCR
THROMBOPHILIA APTT <1.9 females
[HYPER COAGULABLE STATE] Secondary causes of Protein C deficiency:
­ Warfarin therapy
Who should be investigated? ­ Liver disease
­ DIC
­ Venous thrombosis < 40 years
­ Arterila thrombosis < 30 years Secondary causes of Protein S deficiency:
­ Recurrent thrombosis (more than one ­ Liver disease
episode) ­ DIC
­ Thrombosis at unusual sites (Buddchiari, ­ Nephrotic syndrome
Portal vein, Sagital sinus)
Page 64 of 72
­ Pregnancy computed tomography (spiral CT). Advantages are
clinical equivalence, better access for patients and the
Secondary causes of AT-III deficiency: possibility of picking up other lung disorders from the
­ Heparin therapy differential diagnosis in case there is no pulmonary
embolism
­ Nephrotic syndrome
­ DIC Treatment of deep vein thrombosis:
­ Liver disease ­
­ Unfractionated Heparin : intravenous bolus
Symptoms and Investigations: dose 80 u/kg followed by continuous infusion
DVT: leg pain, tenderness, swelling (edema), increased at a rate of 18 u/kg/hr., then adjust dose
warmth, and change in skin color (redness) according to APTT.
The presence of deep venous thrombosis may be
­ Alternatively, start LMW heparin, dose per
detected by:
body weight and no need for laboratory
• venography of the leg
monitoring.
• doppler ultrasound - Start coumadin within 24 hour of starting heparin,
• plethysmography monitor INR that need to be maintained at 2 – 3. Once
• positive D-dimer test this range achieved for at least 2 days you may
discontinue heparin. Warfarin therapy often requires
PE frequent dose adjustment and monitoring of the INR.
Signs of PE are sudden-onset dyspnea, tacypnea, chest In proximal DVT and PE, INR between 2.0 and 3.0 are
pain of "pleuritic" nature, cough hemoptysis, and in generally considered ideal. If another episode of DVT
severe cases, hypotension, shock, loss of or PE occurs under warfarin treatment, the INR
consciousness, and death. Although most cases have window is often increased to 3.0-4.0 (unless there are
no clinical evidence of deep venous thrombosis in the contraindications).
legs, findings that indicate this may aid in the ­ Continue with warfarin therapy for at least 6
diagnosis. months. or "lifelong" if there have been
ECG may occasionally show right heart strain (the previous DVTs or PEs.
"S1Q3T3 pattern).
The Gold standard for diagnosing pulmonary Treatment outcome:
embolism (PE) is still pulmonary angiography. In most - DVT recurrence: during heparin therapy< 5 %
of the cases, however, when PE is suspected on the during warfarin therapy < 2 %
basis of shortness of breath and chest pain, the during the first year 2–5%
following scans may confirm the presence of an
­ Most recurrence have hereditary or acquired
embolus:
thrombophilia
"Ventilation-perfusion scan" (or V/Q scan), which
shows that some areas of the lung are being ventilated ­ 20 % of thrombophilic patients have recurrence in
but not perfused with blood (due to obstruction by a 1 year
clot). Increasingly, the V/Q scan is being replaced with ­ > 50 % of thrombophilic patients have recurrence
in 5 years

Page 65 of 72
ANTITHROMBOTIC AGENTS
2. IV.Bolus: 5,000 – 10,000 u Q 4th P.T.T just prior to
HEPARIN next dose should be therapeutic
Structure – A potent organic acid which is a
mixture of sulphate containing mucopolysacchiarides 3. Prophylaxis: 5,000 u q.8h or q.12 h., sc for
with molecular weights between 3,000 – 30,000 strong surgical cases start on admission, or 2h.
negative charge, prepared from animal tissues. preoperative and continue q. 8-12 h. postoperative;
Mechanism of Action – combines readily peak P.T.T 2-3 hours after injection should be
with plasma proteins, notably Antithrombin III below the therapeutic range.
(heparin co-factor). Heparin markedly accelerated the Note: The P.T.T is also prolonged by warfarin
inhibition action of AT III on thrombin and Factors derivates. Perform a CBC count every three days while
XII, XI, IX and X. Heparin also activates lipoprotein on heparin to avoid heparin induced anemia or
lipase. It does not cross the placenta. thrombocytopenia.
Administration – continuous intravenous
infusion or intermittent bolus and subcutaneous (NOT
INTRAMUSCULAR) Low-Molecular-Weight Heparin
Onset – I.V. – immediate maximum effect; s.c.
– gradual effect and maximum 2-3 hours Low-molecular-weight heparin is a relatively new
Duration of Effect – mean half life is 60 class of anticoagulants that are derived from standard
minutes in normal; shortened with extensive heparin through either chemical or enzymatic
thrombosis and pulmonary embolism; prolonged depolymerization. Whereas standard heparin has a
slightly in renal failure. molecular weight of 5,000 to 30,000 daltons, low-
Degradation and Excretion – Degraded in molecular-weight heparin ranges from 1,000 to 10,000
the liver in large doses, some is taken up by R.E.S and daltons, resulting in properties that are distinct from
a small amount may remain unbound in the plasma. those of traditional heparin.
Antidote – Protamine sulphate – I.V. – slowly;
effective immediately 10 mg for 1,000 units Heparin;
Mechanism of Action:
100 mg. maximum at one time; decrease dose
according to time elapsed since Heparin given to allow
for half life decline in the infused heparin. -Binding of LMW heparins to antithrombin accelerates
Major Uses – thromboplebitis; pulmonary the inhibition of coagulation factor Xa and thrombin
embolism; artificial pumps and filters (i.e renal dialysis about 1000-fold
and bypass pumps) -Unlike unfractionated heparin, which blocks thrombin
Contraindications – active bleeding sites – and factor Xa equally well, LMW heparins primarily
i.e. GI tract; known hemostatic defect i.e. factor VIII block coagulation factor Xa
deficiency; recent CNS surgery; thrombocytopenia;
severe hypertension.
Problems – Bleeding; uncommon, rarely fatal; Advantages of Low-Molecular-Weight
occasionally produces thrombocytopenia; antiplatelet Heparin
drugs enhance bleeding and should not be used (i.e.
Aspirin) The clinical advantages of low-molecular-weight
Monitoring – Heparin prolongs various heparin include predictability, dose-dependent plasma
clotting test times (anticoagulant effect) in direct levels, a long half-life and less bleeding for a given
proportion to the amount of heparin in the sample antithrombotic effect. Furthermore, immune-mediated
tested. P.T.T (activated partial throboplastin time); thrombocytopenia is not associated with short-term use
therapeutic range is 60-80 seconds; dependent on of low-molecular-weight heparin, and the risk of
laboratory method and should be approximately 1½ - 2 heparin-induced osteoporosis may be lower than the
times control range. risk with the use of standard heparin. Low-molecular-
Dosage and Control: weight heparin is administered according to body
1. Continous I.V: 5,000 u bolus I.V, then 280 weight once or twice daily, both during the high-risk
u/hours in 5% d/W or saline, use hourly mini drip period when prophylaxis for DVT is recommended
chamber (syringe pump better if available); check and also when waiting for oral anticoagulation to take
P.T.T 2 x daily and adjust accordingly; if too low, effect in the treatment of DVT. The activated partial
give 5,000 u bolus and increase drip rate. thromboplastin time (aPTT) does not need to be
monitored, and the dosage does not need to be
Page 66 of 72
adjusted. Since low-molecular-weight heparin is given a) mild INR increase – 1-2 mg Vit K po or SC
subcutaneously, outpatient administration by the b) For severe hypoprothrombinemia alone – 5-10 mg
patient, with or without the assistance of a visiting Vitamin K1
nurse or family member, is both possible and cost- c) If accompanied by bleeding – 25 mg. Vitamin K1
effective. + plasma
Major uses – Prophylaxis + treatment of –
Preparation (trade name): thrombophlebitis, pulmonary embolism, embolic heart
disease i.e rheumatic mitral valve disease
Treatment – follow Heparin therapy for the
• Dalteparin sodium (Fragmin)
above.
• Enoxaparin sodium (Clexane) Contraindication – active bleeding sites, specially
• Nadroparin calcium (Fraxiparin) GI; known hemostatic defet – i.e. Factor VIII
• Tinzaparin sodium (Innohep) deficiency; recent C.N.S surgery, unreliable patient or
no laboratory control; liver and kidney disease
WARFARIN DERIVATES: (COUMADIN) (relative); pregnancy; thrombocytopenia.
Structure – A water-soluble derivative of Problems
Coumeric Acid; 1. Enhance the Effect of Coumadin – decreased
Mechanism of Action – not clear; bound to Vitamin K absorption, i.e. malabsorption,
albumin; acts in the liver to block the synthesis of the pancreatic disease; decreased gut production of
Vitamin K dependent Factors (II, VII, IX, X), and, vitamin K, i.e. broad spectrum antibiotics;
therefore, interfere with both the intrinsic (II, IX and displacement of albumin binding site, i.e.
X) and extrinsic (II, VII, X) system – may block an phenylbutazone.
enzyme regulating Vitamin K metabolism or may be a 2. Decrease the Effect of Coumadin – (require
competitive inhibitor of Vitamin K for specific sites on increased dosage of Coumadin) liver micro-
regular protein. enzyme inducer, i.e. barbiturates.
Administration – by mouth; Dosage and Control – 10 mg p.o. for 2-3 days
Onset and Duration of Effect – does not according to INR, and then a maintenance dose
destroy circulating factors but prevent production; according to INR (usually about4 mg daily). The
therefore dependent on half-life of the above factors warfarin dose should be administered at a consistent
which range from 3 or 4 hours to 72 hours, time of the day and the INR blood sample should be
prothrombin time may be therapeutic in 3-4 days but drawn at a consistent time of day.
full therapeutic effect requires 3-5 days. INR, therapeutic range:
Degradation and Excretion – not absolutely a. Simple Venous Thromboembolism INR of 2.0 –
clear, probably the liver; degradation products excreted 3.0
in the urine. b. Arterial and Valvular Prosthesis INR of 3.5 – 4.5
Antidote – Vitamin K1 (p.o. or I.V. ; takes a Frequency should be daily initially, lengthening to one
minimum of 8-12 hours for effect weekly or at the most every other week once well-
controlled.

Page 67 of 72
PROBLEMS FOR TUTORIALS
factor concentrates after thorough training by the
hemophilia center staff. Now he is bright, very active
HEMOSTASIS / THROMBOSIS boy who seems to seek out precisely those games and
1. A YOUNG BOY WITH SWOLLEN KNEE stunts from which his parents have tried to discourage
his participation. His infusions are at times difficult
A 1-year old infant was noted to ooze intermittently because he squirms. To avoid them he does not
from a circumcision, and his thigh had become swollen consistently inform his parents until pain is severe, or
and tense following a vitamin K injection. He was born he begins to limp noticeably. A re-view of a year's
at term to a 26-year old gravida IV para III woman infusion records reveals he had 68 bleeding events
following an uneventful pregnancy. Delivery was requiring 78 infusions, mostly involving knees and
vaginal over a small midline episiotomy, and he cried ankles. A disproportionate number involved his right
vigorously. The mother's health had always been ankle and were repeated the next day because of
excellent, and she denied any abnormal bleeding or persistent pain and swelling. For the most part, pain
bruising. Her family history was likewise unrevealing. began in school and his parents infused him at home,
Before the infant was transferred to the neonatal often hours later. In the clinic he appeared to walk
special unit, the following studies returned: normally, but his right ankle was swollen, nontender,
but boggy to palpation. Its range of motion was
Hemoglobin 167g/L slightly decreased. An X-ray revealed moderate
Hematocrit 52% demineralization of the adjacent bones but no
Platelet count 339 x 109/L narrowing of the joint spaces, subchondral cyst
White blood cell count 19.4 x 109/L formation, or bony spurs. Pertinent laboratory studies
Prothrombin time (PT) 30 sec (nl 10-16 sec) showed only mildly elevated ALT: 65 U; AST: 72U
Activated partial thromb- 115sec (nl 31-55 sec) (normal levels less than 40 U/mL)
oplastin time (aPTT)
1. What do you make of this follow up
On examination, the infant was pink, easily aroused, information and what would you recommend
and had good muscle tone. Bright red blood discolored to this patient
the circumsion dressing. The left thigh was swollen, its 2. What further tests would you order
circumference measurably larger than the right. A
repeat hematocrit several hours later was 37%, the
2. A BLEEDING YOUNG WOMAN
aPTT was 105 sec and the PT was 16 sec. Coagulation
studies were as follows: factor XII: 60% (nl 13-93%);
Brenda, a 14-year-old girl, came to the emergency
factor XI: 45% (nl 10-66%); factor IX: 39% (nl 25-
room with 18 days of profuse vaginal bleeding and
91%); factor VIII: less than 1%(nl 40-180%); factor
with pallor and weakness. This was her second
XIII screen: normal; fibrinogen: 2.55 g/L (nl 1.67-4.0)
menstrual period, and because it was painful, she had
taken several aspirins. She denied any recent sexual
Questions:
activity and said she had always been in perfect health.
1. What is your interpretation of the Lab
A age 12, however, she had bled so much after a tooth
findings?
extraction that her dentist had to pact and suture the
2. How the presentation is different in those with
bleeding site. Like one of her younger brothers, she
Platelet dysfunction?
bruised easily. Her father had once required a blood
3. How would you manage this patient?
transfusion after a tonsillectomy but had an uneventful
4. What would be your advice for the parents ?
postoperation course following a herniorrhaphy.
Two months later this mother brought her baby to the
Her blood pressure was 110/60 recumbent and 90/50
emergency room because he had cried inconsolably all
sitting up; resting pulse was 100. She was dizzy and
night. By that age he had attained the expected
anxious when she was upright. Her abdomen was soft
milestones and was generally an active, happy baby.
and nontender; and no masses were felt. On bimanual
The mother denied that he had any fever but noted that
pelvic examination, the uterus was midline, anteverted
he did not use one arm at all. Any attempt to move it
and normal in size; the adnexae were soft and
caused him to cry. On examination, none of his joints
nontender. There was much blood flowing from an
or muscles was swollen or tender; but trying to bend
intact cervical os to the vagina.
one elbow made him cry. Within an hour of infusing
factor VIII concentrate, all symptoms subsided.
The laboratory findings were as follows:
Hemoglobin 60g/L
At age 6 years, the patient was seen at an annual
MCV 85 fL
multidisciplinary clinic for routine evaluation. At age
WBC 16.4 x 109/L, with normal differential
5, his parents began home care, injecting him with
Page 68 of 72
aPTT 39 sec (nl 33-27 sec) Thrombin Time (TT): 20 (control 14 s)
PT 14 sec Fibrinogen: 1.7 g/L (N2 – 4)
Pregnancy test negative D- dimer 4000 (N < 500)

She was admitted to the hospital and received one unit Questions:
of packed red blood cells and two 25-mg doses of 1. Comment on the results of the tests? Comment
conjugated equine estrogen, with some improvement. on the peripheral blood film below.
She later received a progestational agent. She was 2. What is your presumptive diagnosis?
discharged on medroxyprogesterone for 10 days. On 3. How would you manage this patient?
next visit, a bleeding time was 12 min (nl less than 9
min), and the following clotting studies were obtained:

aPTT 38.5 sec (nl 27-37 sec)


PT 14 sec (nl 11-14 sec)
Factor XII 60% (nl 52-164%)
Factor XI 72% (nl 67-127%)
Factor IX 73% (nl 55-163%)
Factor VIII:C 38% (nl 40-149%)
Factor VIII:Ag 35% (nl 40-149%) 4. A BLEEDER BABY BOY
Ristocetin cofactor 45% (nl 50-150%) A 1-year-old boy was referred from a dentist to the
emergency unit after failing to control a gum bleed,
Questions: which took place at a new teething site. The mother
1. What is the clinical presentation suggests mentioned that the child also bled profusely following
2. How do you interpret the lab data a circumcision, which was done at the age of two
3. How would you manage this patient months. Apart from these two incidents, the child has
4. Comment on inheritance mode been generally well and growing normally. The boy
was the second child in the family, has a 5 years old
sister who has no medical problem. The parents were
3. FEVER AND COAGULOPATHY non- consanguineous but the mother mentioned that
3 years old boy was brought to the emergency unit by she has a brother who is a “bleeder”.
his mother. The child has been unwell for the past three
weeks with fever and loss of appetite. He has been On examination, the child was healthy, a febrile,
diagnosed to have an upper respiratory tract infection oozing blood from a teething site. He also had few
by the family doctor and was given Amoxill but bruises on his shins. Examination of the chest,
without any improvement. Two days before abdomen was normal
presentation, the mother noticed the appearance of
pinpoint reddish spots on his shins and arms. Investigations revealed:
The boy has been generally well, received all his WBC: 6.9 x 109 /L
vaccinations and does not have any known illness or Hb. 120 g /L
allergies. Platelets: 345 x 109 /L

On examination, the boy was fully conscious but PT : 15 s (N 10 – 14s)


irritable. Temp: 39ºC. There was a big bruise on his APTT : 55 s (N 25 - 35 s)
forehead and petechial hemorrhages on his legs, arms Fibrinogen 3.2 g/L
and abdomen. Examination of the neck revealed the D-dimer: 200 μg/ml (N <500)
presence of two small lymph nodes in the cervical Bleeding time 5 min (N 2-7)
region.
Examination of the chest and heart was normal. Question:
Examination of the abdomen was normal apart from 1. What is your presumptive diagnosis?
feeling a splenic tip. 2. How would you prove this diagnosis?
3. How would you manage this patient?
Investigations showed:
WBC: 40 x 109 /L 5. BLEEDING GUMS AND BRUISING
Hb.: 105 g/L Normal Male: 140-180, Female: 120- A 23- year woman came to the emergency room
140 complaining of bruises over the body and bleeding
Platelets: 25 x 109 /L N 150 - 400 gums for the preceding 36 hours. The patient never has
never had easy bruisability, epistaxis, gum bleeding, or
Prothrombin time (PT): 20 sec (control 13 s) / INR:2 bleeding into the joints. Menarche was at age 13 and
APTT: 60 (control 38 s)
Page 69 of 72
menses have been normal. The last menstrual period Examination reveals a respiratory rate of 26/min, HR
was 16 days prior to admission. And was normal in 110/min, BP 110/60. The right calf was plethoric,
amount and duration. The patient had several dental swollen and tender. The reminder of the examination,
extractions at age 17 and oozed for 12 hours thereafter. including the chest examination, was normal
There has not been excessive bleeding with minor
trauma. She has not had surgery or pregnancies. A Doppler ultrasound of the right leg revealed
Family history is negative for bleeding disorders. thrombus in the popliteal and superficial femoral vein.

On physical examination the patient is a well- Investigations:


developed young woman without pallor or jaundice. CBC
Vital signs are normal. Fundoscopic examination is HB 121 g/L
unremarkable. There is oozing of the gingivae with RBC Count 4 x 1012/L
several small hemorrhagic bullae on the buccal Hct 0.41
mucosa. There is an ecchymoses and oozing in the MCV 85 fl
right antecubital area secondary to venipuncture and MCH 31 pg/L
there are several ecchymoses and multiple petechiae MCHC 340 pg/L
on both lower extremities. There is no palpable RDW 13
adenopathy or splenomegaly. The remainder of the WBC 12 x 109/L
examination is normal. Platelets Count 280 x 109/L

Laboratory Data Coagulation Screen


APTT 26 seconds
PT 13 / 12 –second control INR: 1 PT 11.4 seconds
APTT / 33 –second control 30 second INR 1.0
WBC 6x109/L – normal differential Fibrinogen 2.5 g/L
HB 125 mg/L
Platelets 8,000/mm3 Specific Tests
Test Results Normal
Questions:
1. What dose the combination of bruising and oral ATIII 0.9 0.75 – 1.15
mucosal bleeding implies? What dose the negative Protein S (free) 0.75 0.50 – 1.50
clues in the history suggests? Protein S (Total) 1.10 0.75 – 1.20
2. Based on the history what is your differential Protein C Activity 0.12 0.50 – 1.75
diagnosis? Protein C Antigen 0.15 0.65 – 1.50
3. Looking at the CBC results, what is the next APCR Ratio 2.8 >2
immediate test you would ask for? What dose the
normal INR and APTT results imply? Questions
4. Considering the low platelets count, what relevant 1. Discuss the investigations of suspected deep
piece of history you would ask the patient? And venous thrombosis and pulmonary embolism
what are the other relevant tests you would ask
for? 2. What conditions or situations may have
5. What is your differential diagnosis of predisposed to this event?
thrombocytopenia in this patient? Given the most 3. What are the causes of hypercoagulable state?
likely diagnosis how would you manage this 4. Discuss the management of this patient under the
patient? following headings:
­ Anticoagulant therapy
­ Monitoring therapy
6. A YOUNG PATIENT WITH SUDDEN ­ Family study
SHORTNESS OF BREATH
A 24-year-old female university student presented to
7. OFTEN MISSED BLEEDING PROBLEM
the emergency room complaining of a sudden onset
A 34-year old woman was seen by a gynecologist for
left sided pleuritic chest pain and Shortness of breath
tubal ligation. During the interview, she described a
of one-hour duration. She admitted to having a painful
lifelong problem with easy bruising with minimal
swollen right calf of several days duration. She has
trauma. She also described the frequent nosebleeds as
recently returned from a summer holiday in Florida.
a child and heavy menstrual periods, particularly in the
She had been taking the oral contraceptive pills for 6
last two years.
months. She has a younger brother with a history of
deep venous thrombosis. Her father has taken warfarin
for many years. Her mother and 3 siblings are well.
Page 70 of 72
Physical examination was entirely normal, except c. If the bleeding progressed is there anything
for a few small bruises on her legs. There were no you could prescribe to correct the coagulation
petechiae. problem more quickly?
Laboratory Investigations
Hemoglobin 130 g/L
WBC 7.9 x 10 9/L 9. ALCOHOLIC AND COAGULOPATHY
Platelet count 280 x 109/L A 53 year old formed alcoholic and previously been
P.T.T 47 seconds investigated during a G.I. bleed and was found to have
Thrombin Time 9/9 seconds cirrhosis and esophageal varices. She was brought to
Bleeding time 12 minutes the Emergency Room by friends who had found her
INR 1.0 lying on the floor of her apartment.
Questions: In the Emergency Room she vomited "coffee-ground"
1. List 2 additional points in the history which you material, and had a melena stool. On examination, her
would require pulse was 100/minute and blood pressure was 106/70
2. What is the most likely diagnosis? supine. She had mild scleral icterus, spider nevi on her
3. How is this disorder inherited? face and chest, moderate splenomegaly (4 cm below
4. The patient is booked for tubal ligation left costal margin) and mild ascites. The liver was
a. What precautions would you advise palpable.
her about daily lifestyle? Laboratory Investigations:
b. Which of the following would be most Hemoglobin 84 g/L
appropriate before the operation or MCV 100 fL
post-operatively MCHC 320 g/L
i. FFP WBC 3.6 x 109/L
ii. Cryoprecipitate Platelet count 70 x 109/L
iii. Factor VIII concentrate INR 4.0 seconds
iv. None of the above P.T.T 54 seconds
Questions:
8. POST OPERATIVE DVT 1. List 3 possible causes for the anemia?
Four days after a left nephrectomy, a 45 year old man 2. List 3 possible causes for the thrombocytopenia.
developed a deep vein thrombosis in his left leg. He 3. Explain the elevated INR and P.T.T.
had no respiratory symptoms. The foot of the bed was 4. The intern orders:
elevated and he was given IV injection of Heparin, a. bleeding time – is this a reasonable
5000 units, followed by a continuous IV heparin order
infusion at 1000 units per hour. Later on his 5. Discuss your management regarding
anticoagulation was changed to coumadin orally. a. Initial I.V. order
b. The anemia
Questions: c. The thrombocytopenia
1. List 3 techniques which might have helped prevent d. The coagulation defect (2 points)
the DVT
2. The intern orders "Coumadin 10 mg daily", what is 10. FEVER POST SPLENECTOMY
wrong with this? A 60 year old man was admitted to hospital with a
3. The intern orders "INR daily", what is wrong with three day history of fever, rigors, cough and yellow
this order? sputum and pleuritic left lower chest pain. He had been
After 10 days of coumadin therapy, he previously been healthy except for a previous
complained of nausea, and faintness. He splenectomy 12 years before following a traumatic
had a large melena stool. He was pale, rupture of his spleen in a farm accident. On
clammy and perspired profusely. Pulse examination he was very ill with a high fever and signs
was 110/min and regular. Blood pressure of pulmonary consolidation in the lower lobe of his left
was 95/96 with significant postural drop. lung. Pulse was 140/minute and blood pressure 130/60.
Hemoglobin was 110 g/L., (down from his He had many petechiae and bruises on his trunk and
last known Hb of 130 g/L. three days extremities.
earlier) INR = 6.1 Laboratory Investigations:
4. Describe your management under the following Hemoglobin 116 g/L
headings: WBC 15 .5 x 109/L
.68 neutrophils
a. Amount of coumadin you would prescribe?
.16 bands
b. What medication would you order, how much .02 metamyelocytes
and by what route?
.01 myelocytes

Page 71 of 72
.13 lymphocytes 5. After three weeks there has been little response to
RBC fragmentation ++ treatment, and splenectomy is advised; what
Howell Jolly bodies + chance is there of complete remission of the
Platelet count 26 x 109/L thrombocytopenia with splenectomy? – 50%, 80%,
Reticulocyte count 60 x 109/L 100%
INR 3.5 seconds 6. What is the main long-term risk of splenectomy?
P.T.T 80 seconds What could you do in an attempt to avoid this
Thrombin Time 18/9 seconds complication?
Fibrinogen 0.5 g/L What is the role of IVIgG in this disease?

Questions:
1. Give a brief explanation for:
a. abnormal WBC differential
b. RBC fragmentation
c. Howell-Jolly bodies
d. Thrombocytopenia
e. Normal reticulocyte count
f. Coagulation defects
2. Discuss your management under the following
headings:
a. Thrombocytopenia – type, amount and rate of
blood products; expected response
b. Coagulation defect

11. BLEEDING WITH SEVERE


THROMBOCYTOPENIA

A 25-year old woman with no previous history of


abnormal bleeding or bruising visited her doctor after 3
week history of increasing spontaneous bruising. Her
last menstrual period, two weeks before was normal,
but she did have a prolonged nosebleed two days ago.
Her only medications included multivitamin tablet,
occasional diazepam for sleeping, and aspirin tablets
for headaches (her last aspirin was three days ago). Her
doctor noticed some crusted blood in both nostrils, a
few hemorrhaging bullae on her tongue and buccal
mucosa, and numerous petechaie and bruises on her
extremities. She was afebrile, and appeared otherwise
healthy.

Laboratory Investigations
Hemoglobin 130 g/L
MCV 88 fl
MCHC 340 g/L
WBC 6.5 x 109/L
Differential normal
Platelet count 10 x 109/L

Required Questions:
1. What is the most likely diagnosis?
2. Should the patient be treated at home or in
hospital?
3. What additional test or procedure would usually be
done?
4. What medication would you initially prescribe?

Page 72 of 72

You might also like